You are on page 1of 154

GENERAL

MATHEMATICS

Name:________________________________ Section: ____________________


Teacher: ______________________________Mobile No: __________________
Email:________________________________ Sem./S.Y.:___________________
Introduction

General Mathematics for Senior High School – Core Subject is designed to give
background in general mathematical concepts taught in all programs in senior high
school. The text's primary concepts and procedures are needed to succeed in all senior
high school offerings of the Department of Education.

Students are also encouraged to manipulate a reduced set of general mathematical


formulas. The text provides a comprehensive discussion of the concepts of the General
Mathematics Curriculum as prescribed by the Department of Education, which includes
functions, rational functions, inverse functions, exponential functions, logarithmic
functions, simple and compound interest, simple and general annuities, stocks, and
bonds, and logic. It is intended that the next be adaptable to a one-semester course in
general mathematics following the K to 12 Curriculum of the Department of Education.

Each chapter starts with a list of learning outcomes that need to be met before moving
to the next chapter of the text. The course outline immediately follows it in each
chapter and inspiring quotations by known personalities.

GENERAL MATHEMATICS 2|Page


Table of Contents
Table of Contents……………………………………………………………………..……………………………………..
………..……….3

MELC (Most Essential Learning Competencies) ………………………………………….…. 5

QUARTER I
Lesson 1: Representation of function………………………………………………………..……….………. 15

Pre – Test………………………………………….…………………….…………….……. 15
Lesson 2: Operations on Functions..………………… …………………….……………………. 23
Pre – Test……………………………………………………….…………………………... 23
Lesson 3: Rational Function....……………………………………… ……………… ………….... 30
Pre – Test………………………………………………………………………………….... 30
Lesson 4: Solving Rational Equations and Inequalities………………………………….... 39
Pre – Test……………………………………………………………………………..….... 39
Lesson 5: Exponential Function…..…………………………………………………………….... 43
Pre – Test………………………………………………………………………………….... 43
Lesson 6: Logarithmic Function..……………………………… …………… …………………....48
Pre – Test………………………………………………………………………………...... 48
QUARTER II
Lesson 1: Simple Interest…………………………………………………………………………......55
Pre – Test…………………………………………………………………………………......55
Activity #1………………………………………………………………………………….... 62
Lesson 2: Compound Interest………………………………………………………………….….....65
Pre – Test………………………………………………………… .………………………....65
Activity #1………………………………………………………………………………….....67
Activity #2………………………………………………………………………………….....71
Lesson 3: Simple Annuity…...……………………………………………………………………......74
Pre – Test…………………………………………………………………………….…….....67

GENERAL MATHEMATICS 3|Page


Activity #1………………………………………………………..…………….………...78
Lesson 4: General Annuity…...………………………………………………………….……...100
Pre – Test………………………………………………………… ………………......100
Activity #1………………………………………………………… ……….……….…..109
Lesson 5: Stocks and Bonds..…………………………………………….… ………………….111
Pre – Test……………………………………………………….… ……………….…...111
Activity #1………………………………………………………………………..….....116
Activity #2………………………………………………………………………….......117
Activity #3………………………………………………………………………….......120
Activity #4………………………………………………………………………….…....121
Activity #5…………………………………………………………………………….....124
Lesson 6: Logic………….…...
…………………………………………………………………………........126
Pre – Test…………………………………………………………………………........126
Activity #1…………………………………………………………………………….....128
Activity #2………………………………………………………………………………..130
Activity #3…………………………………………………………………………….....131
Activity #4………………………………………………………………………….…....133
Activity #5…………………………………………………………………………........134
Activity #6…………………………………………………………………………........135
Lesson 7: Truth Tables of the Propositions…….…...……………………………..……....138
Pre – Test………………………………………………………..….………………….....138
Activity #1…………………………………………………………….……………….…..140
Lesson 8: Tautologies, Contradiction and Contingency…….…...………………….…...142
Pre – Test…………………………………………………………..….…………………..142
Activity #1……………………………………………………….….…………………....145
Activity #2……………………………………………………….….…………………....147
Activity #3………………………………………………………………………..…….....150

GENERAL MATHEMATICS 4|Page


General Mathematics
Most Essential Learning Competencies
First Quarter
Gradin Most
g Essential
Period Learning
Learning Resources
Competencie
s
Week  represents  https://www.slideshare.net/swartzje/piecewise-
1 real-life functions-26387725
situations
using
functions,
including
piece-wise
functions.
 evaluates a  https://www.slideshare.net/Dreams4school/relations-
function. and-functions-42999445
 performs  https://www.slideshare.net/swartzje/operations-on-
addition, functions
subtraction,
multiplicatio
n, division,
and
composition
of functions
 solves  https://www.slideshare.net/Dreams4school/relations-
problems and-functions-42999445
involving
functions.
Week  represents  https://www.slideshare.net/zozima/rational-functions-
2 real-life 32324533
situations
using
rational
functions.
 distinguishe  https://www.slideshare.net/pemey13/rational-
s rational equations-and-inequalities
function,
rational
equation,

GENERAL MATHEMATICS 5|Page


and rational
inequality.
 solves  https://www.slideshare.net/pemey13/rational-
rational equations-and-inequalities
equations
and
inequalities.
 represents  https://www.slideshare.net/zozima/rational-functions-
a rational 32324533
function
through its:
(a) table of
values, (b)
graph, and
(c)
equation.
 finds the  https://www.slideshare.net/zozima/rational-functions-
domain and 32324533
range of a
rational
function.
Week  determines  https://www.slideshare.net/zozima/rational-functions-
3 the: (a) 32324533
intercepts;
(b) zeroes;
and (c)
asymptotes
of rational
functions
 solves  https://www.slideshare.net/zozima/rational-functions-
problems 32324533
involving
rational
functions,
equations,
and
inequalities.
Week  represents  https://www.slideshare.net/JerriHarb/inverse-
4 real-life functions-2487153
situations
using one-
to-one
functions.
 determines  https://www.slideshare.net/JerriHarb/inverse-
the inverse functions-2487153

GENERAL MATHEMATICS 6|Page


of a one-to-
one
function.
 represents  https://www.slideshare.net/JerriHarb/inverse-
an inverse functions-2487153
function
through its:
(a) table of
values and
(b) graph.
 finds the  https://www.slideshare.net/JerriHarb/inverse-
domain and functions-2487153
range of an
inverse
function.
Week  solves  https://www.slideshare.net/JerriHarb/inverse-
5 problems functions-2487153
involving
inverse
functions.
 represents  https://www.slideshare.net/jessicagarcia62/
real-life exponential-functions-4772163
situations
using
exponential
functions.
 distinguishe  https://www.slideshare.net/jessicagarcia62/
s between exponential-functions-4772163
exponential
function,
exponential
equation,
and
exponential
inequality.
Week  solves  https://www.slideserve.com/hedva/4-6-m2-solve-
6 exponential exponential-equations-and-inequalities
equations
and
inequalities.
 represents  https://www.slideshare.net/jessicagarcia62/
an exponential-functions-4772163
exponential
function
through its:

GENERAL MATHEMATICS 7|Page


(a) table of
values, (b)
graph, and
(c)
equation.
 finds the  https://www.slideshare.net/jessicagarcia62/
domain and exponential-functions-4772163
range of an
exponential
function.
 determines  https://www.slideshare.net/jessicagarcia62/
the exponential-functions-4772163
intercepts,
zeroes, and
asymptotes
of an
exponential
function.
Week  solves  https://www.slideserve.com/hedva/4-6-m2-solve-
7 problems exponential-equations-and-inequalities
involving
exponential
functions,
equations,
and
inequalities.
Week  represents  https://www.slideshare.net/swartzje/logarithmic-
8 real-life functions-31293288
situations
using
logarithmic
functions.
 Distinguishe  https://www.slideshare.net/swartzje/logarithmic-
s functions-31293288
logarithmic
function,
logarithmic
equation,
and
logarithmic
inequality.
 solves  https://www.slideshare.net/Jomaizildjian/solving-
logarithmic logarithmic-equations-and-inequalities
equations
and

GENERAL MATHEMATICS 8|Page


inequalities.
Week  represents  https://www.slideshare.net/swartzje/logarithmic-
9 a functions-31293288
logarithmic
function
through its:
(a) table of
values, (b)
graph, and
(c)
equation.
 finds the  https://www.slideshare.net/swartzje/logarithmic-
domain and functions-31293288
range of a
logarithmic
function.
 determines  https://www.slideshare.net/swartzje/logarithmic-
the functions-31293288
intercepts,
zeroes, and
asymptotes
of
logarithmic
functions.
Week  solves  https://www.slideshare.net/Jomaizildjian/solving-
10 problems logarithmic-equations-and-inequalities
involving
logarithmic
functions,
equations,
and
inequalities.
Second Quarter

Gradin Most
g Essential
Period Learning Learning Resources
Competencie
s
Week  Illustrates  https://www.slideshare.net/hisema/simple-and-
1 simple and compound-interest-24834757
compound
interests.
 Distinguishe  https://www.slideshare.net/hisema/simple-and-
s between compound-interest-24834757

GENERAL MATHEMATICS 9|Page


simple and
compound
interests.
Week  Computes  https://www.slideshare.net/hisema/simple-and-
1-2 interest, compound-interest-24834757
maturity
value,
future
value, and
present
value.
 solves  https://www.slideshare.net/hisema/simple-and-
problems compound-interest-24834757
involving
simple and
compound
interests.
Week  Illustrates  https://www.slideshare.net/KrystenAmoranto/simple-
3-4 simple and annuities-50463135
general
annuities.
 Distinguishe  http://www.maxonlinemath.com/math12/jdinvx/
s between genann.htm
simple and
general
annuities.
 Finds the  https://www.investopedia.com/retirement/calculating-
future value present-and-future-value-of-annuities/
and present
value of
both simple
annuities
and general
annuities.
 Calculates  https://www.investopedia.com/retirement/calculating-
the fair present-and-future-value-of-annuities/
market
value of a
cash flow
stream that
includes an
annuity.
 Calculates  https://www.slideshare.net/KrystenAmoranto/simple-
the present annuities-50463135

GENERAL MATHEMATICS 10 |
Page
value and
period of
deferral of a
deferred
annuity.
Week  Illustrate  https://www.slideshare.net/pjgrethel/stocks-bonds
5 stocks and
bonds.
 Distinguish  https://www.slideshare.net/pjgrethel/stocks-bonds
between
stocks and
bonds.
 Describe  https://www.slideshare.net/pjgrethel/stocks-bonds
the different
markets for
stocks and
bonds.
 Analyze the  https://www.slideshare.net/pjgrethel/stocks-bonds
different
markets for
stocks and
bonds.
 Interpret  https://www.slideshare.net/pjgrethel/stocks-bonds
the theory
of efficient
markets.
Week  Illustrates  https://www.slideshare.net/KrystenAmoranto/simple-
6 business annuities-50463135
and
consumer
loans.
 Distinguishe  https://www.slideshare.net/KrystenAmoranto/simple-
s between annuities-50463135
business
and
consumer
loans.
 Solves  https://www.slideshare.net/KrystenAmoranto/simple-
problems annuities-50463135
involving
business
and
consumer
loans

GENERAL MATHEMATICS 11 |
Page
(amortizatio
n,
mortgage).
Week  Illustrates  https://www.slideshare.net/AdilAslam4/chapter-1-
7 and propositions-in-discrete-mathematics
symbolizes
propositions
.
 Distinguishe  https://www.slideshare.net/AdilAslam4/chapter-1-
s between propositions-in-discrete-mathematics
simple and
compound
propositions
.
 Performs  https://www.slideshare.net/AdilAslam4/chapter-1-
the different propositions-in-discrete-mathematics
types of
operations
on
propositions
.
Week  Determines  https://www.slideshare.net/AdilAslam4/chapter-1-
8 the truth propositions-in-discrete-mathematics
values of
propositions
.
 Illustrates  https://www.slideshare.net/AdilAslam4/chapter-1-
the different propositions-in-discrete-mathematics
forms of
conditional
propositions
.
Week  Illustrates  https://www.slideshare.net/forwardblog4u/
9 different propositional-logic-14203172
types of
tautologies
and
fallacies.
 Determines  https://www.slideshare.net/forwardblog4u/
the validity propositional-logic-14203172
of
categorical
syllogisms.

GENERAL MATHEMATICS 12 |
Page
 Establishes  https://www.slideshare.net/forwardblog4u/
the validity propositional-logic-14203172
and falsity
of real-life
arguments
using logical
propositions
, syllogisms,
and
fallacies.

GENERAL MATHEMATICS 13 |
Page
1 QUARTER
st

GENERAL MATHEMATICS 14 |
Page
Lesson 1: Representation of Function

What to Know?

What you can learn and do in this lesson:

 Represent real-life situations using functions, including piece-wise function.


 Represent real-life situations using one-to-one function.

Pretest

Instruction: Determine the following types of relation are they function or not.

Relations Answer
1. {(1,2), (2,5), (3,10), (4, 17)}
2.

Input Output

1 4

2 5

3 6

3. x 2+ y 2=4

GENERAL MATHEMATICS 15 |
Page
4.

5.

All the sets of numbers and graphs in the above activities are called
relations, and the connections of their relations between domain and range are called
function.

Let’s Learn This!

A
Relation
is a set of

These are the types of relation between domain and range.

GENERAL MATHEMATICS 16 |
Page
Table 1.1

Table 1.2

Table 1.3

A Function is a relation in which an element of the domain corresponds to


exactly one element of the range.

As we can see from the table above, only tables 1.1 and 1.3 we say as
function. Why is one-to-many not a function? Simply because the domain has
corresponded to one or more elements of the range.

GENERAL MATHEMATICS 17 |
Page
Take Note

Table 1.4

Table 1.4 is not a function and not a relation because one of the elements in
the domain does not correspond to the element of the range. That is why it can be told
neither a function nor a relation.

Examples:

Instructions: Determine each problem to know what type of relation and identify if
they are function or not.

Problem Relation Function


1. {(1,2), (3,4), (5,6), (7,8)} One – to - One Yes
2. {(2,4), (2,6), (2,8), (2,10)} One – to - Many No
3. Many – to – One Yes
X -3 -2 -1
Y 1 1 3

Many – to – One Yes

4.

GENERAL MATHEMATICS 18 |
Page
5. One – to – One Yes

Take note!

When finding the domain and range of a function involving a radical with an
even index: Radicand must be non-negative. Hence, the radicand must be greater than
or equal to zero.

Activity 2:

GENERAL MATHEMATICS 19 |
Page
Guide Questions
1. How can you identify the graph if it is a function or not?
2. How can you plot the coordinates from the table into a line and identify if it is a
function or not?
3. Is there any types of functions?

Not every graph x-y plane represents a function. The vertical line test is
the simplest way of determining whether or not a graph represents a function.

Vertical Line Test – a graph represents a function if no vertical line intersects the graph in
more than one point.

Examples:

In terms of function, we need to know the types of functions in applying in


mathematical expressions.

1. Constant Function f ( x )=C ; C ≠ 0

2. Linear Function f ( x )=bx+ c ; b ≠ 0

3. Quadratic Function 2
f ( x )=a x +bx + c ; a ≠ 0

4. Polynomial Function n
f ( x )=a x + b x
n−1
+…+ c

5. Rational Function g( x )
f ( x )= ; h(x )≠ 0
h( x)

6. Exponential Function n
f ( x )=x ; n=any real numbers

GENERAL MATHEMATICS 20 |
Page
7. Absolute Value Function f ( x )=¿ x ∨¿

8. Greatest Integer Function f ( x )=[ x]

Examples:

Instructions: Identify what type of functions below.

1. f ( x )=22→ Constant Function

2. f ( x )=x 2 + 4 x+ 4 → Quadratic Function

3. f ( x )= [ x +1 ] → Greatest Integer Function

4. f ( x )=| x−5|→ Absolute Value Function

2
x −4
5. f ( x )= → Rational Function
x +1

Piecewise Function
In the previous discussion of this lesson, a single equation represented each
function. In many real-life problems, functions may be represented by a combination of
equations. Such functions are called compound functions. For example, the compound
function is given by

f ( x )=
{x 2−2x −2x+,if1, ifx ≤1x >1
2

is defined by two equations. One equation gives the value of f ( x ) when x is less than or
equal to 1, and the other equation gives the value of f ( x ) when x is greater than 1.

GENERAL MATHEMATICS 21 |
Page
Piecewise Function or Compound Function – is a function defined by multiple subfunctions,
where each subfunction applies to a certain interval of the main function’s domain.

Example:

1. f ( x )=
{−xx +2+2,if,ifx x≥<00
2

Solution:

To the right of the y-axis, the graph is a line with a slope


of 1 and a y-intercept of 2. To the left of the y-axis, the
function graph is a parabola that opens downward and
whose vertex is (0,2). To sketch the function, you can
lightly draw both graphs. Then darken the portion of the graph that represents the
function.

To find the value of the function when x=−4, use the second equation.

I. One-to-one Function
A one-to-one function is a function in which for each value of y, there is
just one value for x in the domain of f such that y=f (x ).

In other words, f is one-to-one if f (x 1)=f (x 2) implies x 1=x 2

GENERAL MATHEMATICS 22 |
Page
Lesson 2: Operations on Functions

What to Know?

What you can learn and do in this lesson:

 Perform addition, subtraction, multiplication, division, and composition of functions.

Pre – test

Instructions: Perform the indicated operations

a. ( 4 x+3 )+ ( 3 x −2 )

b. ( 2 x 2−3 x−2 )−( 4 x 2 +5 x+3 )

c. ( 2 x−3 ) ( x+ 4 )

d. ( x 2−5 x +2 ) ( 3 x +1 )

Let’s Learn This!

Several functions can work together in one larger function. There are 5 common
operations that can be performed on functions. The four basic operations on functions
are adding, subtracting, multiplying, and dividing. The notation for these functions is as
follows:

Addition ( f +g ) ( x ) =f ( x )+ g ( x )

Subtraction ( f −g ) ( x )=f ( x )−g (x)


GENERAL MATHEMATICS 23 |
Page
Multiplication ( fg )( x )=f ( x )∗g(x )

Division ( fg ) ( x )= gf (x)
(x)

When we do one of these four basic operations we can simply evaluate the two
functions at the value and then do the operation with both solutions

Example 1:

2
f ( x)=x −x−2

g(x )=x +1

find (f + g)(−3)

Evaluate f and g at − 3

2
f (−3)=(−3 ) −(−3)−2 Evaluate f at − 3

f (−3)=9+ 3−2

f (−3)=10

g(−3)=(−3)+1 Evaluate g at − 3

g(−3)=−2

f (−3)+g (−3) Add the two functions together

(10)+(−2) Add

8 Our Solution

The process is the same regardless of the operation being performed.


GENERAL MATHEMATICS 24 |
Page
Example 2:

h( x)=2 x−4 k (x )=−3 x +1 Find(h · k )(5)

Solution:

Evaluate h and k at 5

h(5)=2(5)−4 Evaluate h at 5

h(5)=10−4

h(5)=6

k (5)=−3(5)+1 Evaluate k at 5

k (5)=−15+1

k (5)=−14

h(5)k (5) Multiply the two results together

(6)(−14 ) Multiply

−84 Our Solution

Often as we add, subtract, multiply, or divide functions, we do so in a way that

keeps the variable. If there is no number to plug into the equations, we will use each
equation in parenthesis and simplify the expression.

Example 3:

f (x)=2 x−4

GENERAL MATHEMATICS 25 |
Page
g(x )=x 2−x+ 5

Write subtraction problem of functions Find (f −g)(x)

Solution:

f (x)−g (x) Replace f (x) with (2x − 3) and g(x) with (x2 − x + 5)

(2 x−4 )−( x −x +5) Distribute the negative


2

2 x−4−x + x−5
2
Combine like terms

2
−x + 3 x −9 Our Solution

The parenthesis is very important when replacing f (x)∧g ( x) with a

variable. In the previous example, we needed the parenthesis to know to distribute the
negative.

Example 4:

f (x)=x 2−4 x−5

g(x )=x−5

Find ( fg ) ( x )
Solution:

( fg ) ( x ) Replace f (x) with ( x −4 x−5) and g(x ) with (x−5)


2

(x 2−4 x−5)
To simplify the fraction, we must first factor
(x−5)

( x−5 ) ( x+1 )
Divide out common factor of x − 5
(x−5)

GENERAL MATHEMATICS 26 |
Page
x +1 Our Solution

Just as we could substitute an expression into evaluating functions, we can substitute


an expression into the operations on functions.

Example 5:

f (x)=2 x−1

g(x )=x +4 Write as a sum of functions

Find (f + g)(x2 )

Solution:

2 2
f ( x )+ g( x ) Replace x in f (x) and g(x ) with x 2

2 2
[2( x )−1]+[( x )+ 4] Distribute the + does not change the problem

2
2 x −1+ x + 4
2
Combine like terms

2
3 x +3 Our Solution

Example 6:

f (x)=2 x – 1

g(x )=x +4

Find ( f∗g ) (3 x ) Write as a product of functions

Solution:

f (3 x)g (3 x) Replace x in f (x) and g(x ) with 3x

[2(3x) - 1] [(3x) + 4] Multiply our 2(3 x )

(6 x−1)(3 x+ 4) FOIL

GENERAL MATHEMATICS 27 |
Page
2
18 x +24 x−3 x−4 Combine like terms

2
18 x +21 x−4 Our Solution

The Composition of Functions


The composition of the function f with g is defined by f ∘ g and is defined by the
equation:

( f ∘ g ) ( x ) =f ( g ( x ) )

The domain of the composition function f ∘ g is the set of all x such that
1. x is in the domain of g; and
2. g(x ) is in the domain of f.

Example 7:

a (x)=x 2−2 x +1

b (x)=x−5

Find (a ◦ b)(3)

Solution:

b (3)=(3)−5=−2 This solution is put into a, a (−2)

a (b(3)) Evaluate the inner function first, b (3)

a (−2)=(−2)2−2(−2)+1 Evaluate

a (−2)=4 +4 +1 Add

a (−2)=9 Our Solution

We can also evaluate the composition of functions at a variable. In these problems, we


will take the inside function and substitute it into the outside function.

GENERAL MATHEMATICS 28 |
Page
Example 8:

f (x)=x 2−x

g(x )=x +3

Find (f ◦ g)( x )

Solution:

f (g( x )) Replace g(x) with x + 3

( x +3 )2−(x+ 3) Evaluate exponent

f (x+ 3) Replace the variables in f with (x + 3)

2
( x +6 x+ 9)−( x +3) Distribute negative

2
x + 6 x+ 9−x−3 Combine like terms

2
x + 5 x +6 Our Solution

It is important to note that very rarely is (f ◦ g)( x ) the same as (g ◦ f )( x ) as the following
example will show, using the same equations, but compositing them in the opposite
direction.

Example 9:

f (x)=x 2−x

g(x )=x +3

Find (g ◦ f )(x )

Solution:

GENERAL MATHEMATICS 29 |
Page
g(f (x )) Replace f (x) with x 2−x

2
g( x −x ) Replace the variable in g with ( x 2−x )

2
( x −x )+3 Here the parenthesis don′t change the expression

2
x −x +3 Our Solution

Lesson 3: Rational Function

What to Know?

What you can learn and do in this lesson:

 Represent a rational function through its: (a) table of values, (b) graph, and (c)
equation.
 Find the domain and range of a rational function.
 Determine the (a) intercepts, (b) zeros, and (c) asymptotes of rational functions.

Pretest

True False

1. Every term of a polynomial function is either a


constant, a variable, or a product of constants and
variables.
1
2. The domain of the function f ( x )= is the set of real
x
numbers.
3. The graph of a rational function is a parabola.

GENERAL MATHEMATICS 30 |
Page
4. A rational function is formed by dividing one
polynomial function by another polynomial function.

Let’s Learn This!

Rational functions
A rational function is a fraction of polynomials. That is if p(x) and q(x) are
polynomials, then

p ( x)

q(x)

is a rational function. The numerator is p(x), and the denominator is q(x).

Examples.

3 (x — 5 )
( x — 1)

1
x

3
2x
=2 x 3
1

The last example is both a polynomial and a rational function. Similarly, any polynomial
is a rational function.

In this class, from this point on, most of the rational functions that we will see will
have both their numerators and their denominators completely factored. We will also

GENERAL MATHEMATICS 31 |
Page
only see examples where the numerator and the denominator have no common
factors. (If they did have a common factor, we could just cancel them.)

Implied domains

The implied domain of a rational function is the set of all real numbers

except for the roots of the denominator. That is because it doesn’t make

Example:

The implied domain of

−7 ( x — 2 ) ( x 2+1 )
8( x — 4)( x — 6)

is the set R — {4 ,6 }.

Vertical asymptotes

To graph a rational function, begin by marking every number on the x-axis that
is a root of the denominator. (The denominator might not have any roots.)

Draw a vertical dashed line through these points. These vertical lines are called
vertical asymptotes. The rational function graph will "climb up" or "slide down" the
sides of a vertical asymptote.

Examples:

1
For the rational function , 0 is the only root of the denominator, so the y-axis is the
x
1
vertical asymptote. Notice that the graph of climbs up the right side of the y-axis and
x
slides down the left side of the y-axis.

GENERAL MATHEMATICS 32 |
Page
The rational function

−7 ( x — 2 ) ( x 2+1 )
8( x — 4)( x — 6)

has vertical asymptotes at x = 4 and at x = 6.

x-intercepts

p(x)
The x-intercepts of a rational function (if there are any) are the numbers
q(x)

αϵ R where

p( α)
=0
q (α )

If α is such a number, then we can multiply by q (α ) to find that

p(α )=0 · q(α )=0

In other words, α is the root of p(x ). Thus, the roots of the numerator are exactly the
x-intercepts.

Example:

2 is the only x-intercept of the rational function

GENERAL MATHEMATICS 33 |
Page
−7 ( x — 2 ) ( x 2+1 )
8( x — 4)( x — 6)

In between x-intercepts and vertical asymptotes

When graphing a rational polynomial, first mark the vertical asymptotes and the x-
intercepts. Then choose a number c ∈ R between any consecutive pairs of these marked
points on the x-axis and see if the rational function is positive or negative when x=c .

If it is positive, draw a dot above the x-axis whose first coordinate is c. If it is negative,
draw a dot below the x-axis whose first coordinate is c.

Example:

Let us look at the function

−7 ( x — 2 ) ( x 2+1 )
r (x )=
8(x — 4)(x — 6)

again. The x-intercept of its graph is at x =2, and it has vertical asymptotes at
x=4∧x=6. We need to decide whether r (x ) is positive or negative between 2 and 4 on
the x-axis and between 4 and 6 on the x-axis.

Let us start by choosing a number between 2 and 4, say 3. Then

−7 ( 3 — 2 )( 32+1 )
r (3)=
8 (3 — 4)(3 — 6)

Notice that 7, (3 4), and (3 6) are negative, while 8, (3 2), and (32 +1) are positive.

If you multiply and divide a collection of numbers that are not equal to 0, count how
many negative numbers there are. If there is an even number of negatives, the result
will be positive. If there is an odd number of negatives, the result will be negative. In
the previous paragraph, there are three negative numbers — 7 ,(3 4 ),∧(3 6) — so r (3)<0.

The number 5 is a number that is in between 4 and 6, and

−7 ( 5 — 2 )( 32+1 )
r (5)= >0
8 (5 — 4)(5 — 6)

GENERAL MATHEMATICS 34 |
Page
Far right and far left

Let a x n be the leading term of p(x ) and let bxm be the leading term of q (x) . Recall that
far to the right and left, p(x ) looks like its leading term, a x n.

And far to the right and left, q (x) looks like its leading term, b x m. It follows that the far
right and left portion of the graph of,

p(x)
q(x )

looks like

n
ax
m
bx

and this is a function that we know how to graph.

Example:

2
The leading term of — 7( x — 2)( x + 1) is — 7 x 3, and the leading term of
8( x — 4)( x — 6) is 8 x . Therefore, the graph of
2

−7 ( x — 2 ) ( x 2+1 )
r (x )=
8(x — 4)(x — 6)

looks like the graph of

3
7 x −7
2
= x
8x 8

on the far left and far-right part of its graph.

Putting the graph together

To graph a rational function

GENERAL MATHEMATICS 35 |
Page
p(x)
q(x )

mark its vertical asymptotes (if any). Mark its x-intercepts (if any). Determine whether
the function is positive or negative in between x-intercepts and vertical asymptotes.

Replace p(x) with its leading term, replace q(x) with its leading term, and then graph
the resulting fraction of leading terms to the right and left of everything you have
drawn so far in your graph.

Now draw a reasonable-looking graph that fits with everything you've drawn so far,
remembering that the graph has to climb up or slide down the sides of vertical
asymptotes and that the graph can only touch the x-axis at the x-intercepts that you
already marked.

Example:

Let us graph

−7 ( x — 2 ) ( x 2+1 )
r (x )=
8(x — 4)(x — 6)

First we mark its vertical asymptotes, which are at x = 4 and x = 6, and its x-
intercept, which is at x = 2.

GENERAL MATHEMATICS 36 |
Page
Then we plot points that represent what we had checked earlier for what
happens in between consecutive pairs of x-intercepts and vertical asymptotes: that
r (3)<0∧r (5)>0.

To the left and right of what we have graphed so far, we draw the graph of —7x.

GENERAL MATHEMATICS 37 |
Page
Now we connect what we have drawn so far, making sure our graph climbs up or
slides down the vertical asymptotes and that it only touches the x-axis at the previously
labeled x-intercept.

GENERAL MATHEMATICS 38 |
Page
Lesson 4: Solving Rational Equations and Inequalities

What to Know?

What you can learn and do in this lesson:

 Represent real-life situations using rational functions.


 Distinguish rational function, rational equation, and rational inequality.
 Solve rational equations and inequalities.

Pretest
Complete the table.

a b a∗b Simplified form


A 5 1
x+
5
B 12 1
m−
12
C 3x 4
+4
x
D 10a 1 3
+
2a 5a
E 12m 2 5

3m 2m
F x +2 4
3−
x +2

GENERAL MATHEMATICS 39 |
Page
Let’s Learn This!

A rational equation is an equation that contains one or more rational expressions.


The time t in hours that it takes to travel d miles can be determined using the equation
d
t= , where r is the average rate of speed. This equation is a rational equation.
r

To solve a rational equation, start by multiplying each term of the equation by the least
common denominator (LCD) of all the expressions in the equation. This step eliminates
the denominators of the rational expression and results in an equation you can solve
using algebra.

Example:

18
1. Solve the equation x – =3.
x

Solution:

18
Check x – =3
x

18
Check x – =3
x

GENERAL MATHEMATICS 40 |
Page
An extraneous solution is an equation derived from an original equation that is not a
solution. When you solve a rational equation, it is possible to get extraneous solutions.
These values should be eliminated from the solution set. Always check your solutions by
substituting them into the original equation.

Example:

Solve the equation.

5 x 3 x+ 4
=
x – 2 x –2

Solution:

Check Substitute 2 for x in the original equation.

A rational inequality is an inequality that contains one or more rational expressions.


One way to solve rational inequalities is by using graphs and tables.

Example:

GENERAL MATHEMATICS 41 |
Page
x
Solve ≤ 3 by using a graph and a table.
x−6

Solution:

x
Use a graph on a graphing calculator, Y 1= ∧Y 2=3 .
x−6

The graph of Y1 is at or below the graph of Y2 when x <


6 or when x ≥ 9.

Use a table. The table shows that Y1 is undefined when x = 6 and that Y1 ≤ Y2
when x ≥ 9.

The solution of the inequality is x < 6 or x ≥ 9.

GENERAL MATHEMATICS 42 |
Page
Lesson 5: Exponential Functions

What to Know?

What you can learn and do in this lesson:

 Represent an exponential function through its: (a) table of values, (b) graph, and (c)
equation.
 Find the domain and range of an exponential function.
 Find the intercept, zeros, and asymptote of an exponential function.
 Graph exponential functions.

Pretest

Tell whether each statement is TRUE or FALSE. TRUE FALSE


1. There is an integer x that will make x 2equal to 2 x.
2. Every linear function has an x-intercept.
3. A quadratic function has a vertical asymptote.
4. The graph of y=x intersects the graph of y=− x at (1,
1).
5. 3−2=−9

Let’s Learn This!

1. Exponential functions

Consider a function of the form f (x)=a x, where a > 0. Such a function is called an
exponential function. We can take three different cases, where a=1 , 0<a<1∧a>1.
GENERAL MATHEMATICS 43 |
Page
If a = 1 then

f (x)=1 x=1.

So this just gives us the constant function f (x)=1.

What happens if a> 1? To examine this case, take a numerical example. Suppose that
a=2.

f (x)=2 x

0
f (0)=2 =1

()
1
−1 1 1
1
f (1)=2 =2 f (−1 )=2 = =
2 2

()
2
2
( ) −2 1 1
f (2)=2 =4 f −2 =2 = =
2 4

()
3
−3 1 1
3
f (3)=2 =8 f (−3 )=2 = =
2 8

We can put these results into a table and plot a graph of the function.

This example demonstrates the general shape for graphs of functions of the form
x
f ( x)=a when a> 1.

GENERAL MATHEMATICS 44 |
Page
What is the effect of varying a? We can see this by looking at sketches of a few graphs
of similar functions.

The important properties of the graphs of these types of functions are:

• f (0)=1 for all values of a. This is because a 0=1 for any value of a.

• f (x)>0 for all values of a. This is because a > 0 implies a x >0.

What happens if 0< a<1? To examine this case, take another numerical example.
1
Suppose that a= .
2

()
x
1
f ( x )=
2

()
0
1
f ( 0 )= =1
2

() ()
1 −1
1 1 1
f ( 1) = = f (−1 )= =2
2 2 2

() ()
2 −2
1 1 1
f ( 2 )= = f (−2 )= =4
2 4 2

() ()
3 −3
1 1 1
f ( 3 )= = f (−3 )= =8
2 8 2

We can put these results into a table and plot a graph of the function.
GENERAL MATHEMATICS 45 |
Page
This example demonstrates the general shape for graphs of functions of the form
f ( x)=a when 0< a<1.
x

What is the effect of varying a? Again, we can see by looking at sketches of a few
graphs of similar functions.

The important properties of the graphs of these types of functions are:

• f (0)=1 for all values of a. This is because a 0=1 for any value of a.

GENERAL MATHEMATICS 46 |
Page
• f (x)>0 for all values of a. This is because a> 0 implies a x >0.

Notice that these properties are the same as when a> 1. One interesting thing that you

()
x
1 −x
might have spotted is that f (x)= =2 is a reflection of f (x)=2x in the f (x) axis, and
2

()
x
( ) 1 −x
that f x = =5 is a reflection of f ( x)=5x in the f (x) axis.
5

()
x
1 −x
In general, f (x)= =a is a reflection of f (x)=a x in the f (x) axis.
a

A particularly important example of an exponential function arises when a=e . You


might recall that the number e is approximately equal to 2.718. The function f ( x)=e x is
1∧1
often called ‘the’ exponential function. Since e > <1 , we can sketch the graphs of
e

()
x
x −x 1
the exponential functions f (x)=e ∧f (x)=e = .
e

GENERAL MATHEMATICS 47 |
Page
Lesson 6: Logarithmic Functions

What to Know?

What you can learn and do in this lesson:

 Represent a logarithmic function through its: (a) table of values, (b) graph, and (c)
equation.
 Find the domain and range of a logarithmic function.
 Find the intercept, zeros, and asymptote of a logarithmic function.
 Graph logarithmic functions.

Pre – Test

Tell whether each statement is TRUE or FALSE. TRUE FALSE

y−2
1. If y=3 x +2, then x= .
3
2. If y=3 x , then x=3 y.

3. If 2 x =64 , then x=6 .

GENERAL MATHEMATICS 48 |
Page
4. If b x =1, then x=0 .

5. The inverse of y=x −1 is y=x +1.

Let’s Learn This!

2. Logarithm functions

We shall now look at logarithm functions. These are functions of the form
f (x)=log a x where a> 0. We do not consider the case a=1, as this will not give us a
valid function.

What happens if a> 1? To examine this case, take a numerical example. Suppose that
a=2. Then

f ( x )=log 2 x means 2f ( x )=x .

An important point to note here is that, regardless of the argument, 2f ( x )> 0. So, we shall
consider only positive arguments.

f (1)=log 2 1 means 2f (1)=1 so f (1)=0

f (2)=log 2 2 means 2f (2)=2 so f (2)=1

f (4 )=log 2 4 means 2f (4 )=4 so f (4 )=2

f ( )=log( ) means 2 ( 2 )= 1 =2−1 so


1
1 1 f 1
f ( )=−1
2 2 2 2

f ( )=log ( ) means 2 ( 4 )= 1 =2−2 so


1
1 1 f 1
f ( )=−2
4 4 4 4

We can put these results into a table and plot a graph of the function.

GENERAL MATHEMATICS 49 |
Page
This example demonstrates the general shape for graphs of functions of the form
f (x)=log a x when a> 1.

What is the effect of varying a? We can see sketches of a few graphs of similar
functions. For the special case where a=e , we often write ln x instead of log e x .

The important properties of the graphs of these types of functions are:

• f (1)=0 for all values of a;

• we must have x >0for all values of a.

GENERAL MATHEMATICS 50 |
Page
What happens if 0< a<1? To examine this case, take another numerical example.
1
Suppose that a= . Then
2

()
f ( x)
f (x)=log 1 x 1
means =x .
2 2

()
f ( x)
1
An important point to note here is that, regardless of the argument, > 0. So, we
2
shall consider only positive arguments.

()
x
1
f ( x )=
2

()
f ( 1)
f ( 1 ) =log 1 1 1
means =1 so f ( 1 ) =0
2 2

() ()
f ( 2) −1
f ( 2 )=log 1 2 1 1
means =2= so f ( 2 )=−1
2 2 2

() ()
f (4 ) −2
f ( 4 )=log 1 4 1 1
means =4= so f ( 4 )=−2
2 2 2

f ()
1
2
=log 1
2
1
2
means ()
1
2
f ( 12 )= 1
2
so f ( 12 )=1
() () ( 14 )= 1 =
() ( 14 )=2
2
1 1 1 f 1
f =log 1 means so f
4 2
4 2 4 2

We can put these results into a table and plot a graph of the function.

GENERAL MATHEMATICS 51 |
Page
This example demonstrates the general shape for graphs of functions of the form f
(x )=log a x when 0< a<1.

What is the effect of varying a? Again, we can see by looking at sketches of a few
graphs of similar functions.

The important properties of the graphs of these types of functions are:

• f (1)=0 for all values of a;

• we must have x >0 for all values of a.

An interesting thing that you might well have spotted is that


GENERAL MATHEMATICS 52 |
Page
f (x)=log 1 x is a reflection of f (x)=log x in the x-axis
5
5

and

f (x)=log 1 x is a reflection of f (x)=log x in the x-axis.


2
2

Generally,

f (x)=log 1 x is a reflection of f (x)=log x in the x-axis.


a
a

GENERAL MATHEMATICS 53 |
Page
2 nd

QUARTER

GENERAL MATHEMATICS 54 |
Page
Lesson 1: Simple Interest

What to Know?

What you can learn and do in this lesson:

 Define Simple Interest


 Illustrate simple interest
 Compute interest, maturity value, future value, and present value in a simple
interest environment.

Pretest

I. Express each as a decimal.


1
1. 12% 3. 8 % 5. 200%
2
2. 3% 4. 10.5% 6. 0.75%
II. Answer each.

7. What percent of 24 of 12?

8. what is 25% of 108?

9. 48 is what percent of 60? 30 is 40% of what number?

Let us Learn This!

GENERAL MATHEMATICS 55 |
Page
RULE
Converting Percent to Decimal
To convert a percent to a decimal, drop the percent sign and move the decimal point two places
to the left.

A Percent is the ratio of a number to 100; dividing a number by 100 is the same
as moving the decimal point two places to the left.

When the number in the percent is a whole number, the decimal point is
understood to be stated at the right of the last digit. For example, to convert 12% to a
decimal, drop the percent sign and move the decimal point two places to the left to get
0.12.

To convert
1
2
1 1
( )
% to a decimal, convert first into =0.5 , and then follow the
2 2
1
rule in converting percent into decimal to get %=0.5 %=0.005 .
2

Percent problems are often solved using the formula:

p=r x b

Where p is the portion, r is the rate, and b is the base (the entire amount or the total).
In the statement “25% of 120 is 30,” 25% is the rate, 120 is the base, and 30 is the
portion.

One of the most common uses of this formula involves discounts on sale
items. As indicated at the right, the store gives only the sale price 120.00php and the
discount 25%. Finding the regular price of the item on sale is the same as finding the
answer to the percentage problem: 120 is 75% of what number? (Note: for a discount
of 25%, the amount that you pay for the sale item is 100% minus 25% or 75%.) By
applying the above formula, where 120 is the portion, and 75% is the rate, the base
can be solved as follows:

GENERAL MATHEMATICS 56 |
Page
75 % of x=120 120=75 % of b
0.75 x=120 120=0.75 b
120 120
x= =b
0.75 0.75
x=160 b=160

Hence, the original or regular price of the item on sale at 120.00php is


160.00php.

EXPLORATION
You wish to have 100,000.00php in 5 years. You are making a single deposit for that purpose,
visiting a local bank or any financial institution to obtain information concerning the different accounts
they offer. Has any bank or institution you have visited advised you on what to do? Make a report on this.

Simple interest is charged only on the loan amount called the principal.
Thus, interest on the interest previously earned is not included. Simple interest is
calculated by multiplying the principal by the rate of interest by the number of payment
periods in a year.

To illustrate, Orly invested 100,000.00php to an account that pays a simple


interest of 3% annually. Find the interest earned after 2 years.

Year 1: Interest = 100,000.00php (3%) (1)=3,000.00php

Year 2: Interest = 100, 000.00php (3%) (1)=3, 000.00php

Orly’s investment earned a total of 6,000.00php after two years. Notice from
the above example, the interest earned in Year 1 did not earn any interest. An
alternative computational approach is by multiplying 100,000.00php (principal) by 3%
(rate of interest) by 2 (number of payment periods in a year).

GENERAL MATHEMATICS 57 |
Page
FORMULA
Simple Interest
I =Prt
I I I
( a ) P= (b )r = ( c ) t=
rt Pt Pr
Where:
I = Interest
P = Principal
r = Rate of interest
t = time or term in years or fraction of a year

To find the maturity value, simply add interest to the principal.

Formula
Maturity Value or (Amount or Balance)
A=P+ I or A=P+ Prt or A=P(I + rt)

A = Maturity Value P = Principal I = Interest

The principal P of a loan is also called the face value or the present value of the
loan.

Examples:

1. Teresa borrowed 120,000.00php from her uncle. If Teresa agreed to pay an 8%


annual interest rate, calculate the amount of interest she must pay if the loan period is
(a) 1 year, (b) 9 months, and (c) 18 months.

Solution:

a. We are given P = 120,000php, r = 8% or 0.08, and t = 1 year. Thus,

GENERAL MATHEMATICS 58 |
Page
I =Prt =120,000 x 0.08 x 1=9,600

b. We are given P = 120,000php, r = 8% or 0.08, and t = 9 months or


9
ye a r . Thus,
12

9
I =Prt=120,000 x 0.08 x =7,200
12

c. We are given P = 120,000php, r = 8% or 0.08, and t = 18 months or


18
years .Thus,
12

18
I =Prt=120,000 x 0.08 x =14,400
12

2. If 10,000.00php is invested at 4.5% simple interest, how long will it take to grow to
11,800.00php?

Solution 1:

I
Use the formula t= where P = 10,000.00, r = 4.5% or 0.045 and I = 11, 800.php –
Pr
10,000.00php = 1,800.00php

1,800
t= =4
(10,000)(0.045)

It will take 4 years.

Solution 2:

Use the formula A = P (1 + rt) where A = 11,800.00php P = 10,000.00php, and r =


0.045. Solve for t after substituting values for A, P, and r.

11,800=10,000(1+0.045 t )

11,800=10,000+ 450 t Apply the Distributive Property.

1,800=450 t Subtract 10,000 from each side.

GENERAL MATHEMATICS 59 |
Page
4=t Divide each side by 450.

It will take 4 years.

Note: In the days-in-month method, finding the number of days or finding the desired
date is time-consuming, and it easy to make a mistake along the way. In the day-of-
the-year calendar method, finding the number of days or finding the desired date is less
tedious if you have the day-of-the-year calendar table.

Definition
Ordinary Interest or Banker’s Interest – interest based on a 360-day year.
Exact Interest – interest based on a 365-day year.

Examples:

3. You get a 180-day 200,000.00php loan from a bank at a 10.5% interest. Calculate
interest using (a) 360-day (b) 365-day year.

Solution:

a. 360-day year: I = Prt where, P = 200,000.00php, r = 10.5%, and t =


180 1
=
365 2

1
I =200,000 x 0.105 x =10,500.00 p h p
2

180
b. 365-day year: I =Prt=200,000 x 0.105 x =10,356.16 p h p
365

GENERAL MATHEMATICS 60 |
Page
Note: A 360-day year is favorable for the lender, while a 365-day year is favorable for
the borrower.

Steps of Partial Payments


1. Calculate interest on principal from date of the loan to date of first payment.
2. Remainder of the payment = amount paid – Interest Portion
3. New Balance = Previous Balance – Principals remainder (portion) of payments
4. In more than one partial payment, calculate interest on the new balance from the previous
payment date to the next payment date. Perform steps 2 and 3.
5. at maturity, calculate interest on the last partial payment. Add this to the new balance to
compute the total final payment due.

Example:

4. A loan of 200,000.00php was made from a bank that charges a 9% interest rate and
should be repaid after 90 days. If payment of 80,000.00php was made after 20 days
and the balance on the 90th day, calculate the amount of interest, principal paid for
each payment, and the total amount paid.

Solution:

20
Payment on Day 20: P = 200,000.00php, r = 9%, and t=
365

20
I =Prt=200,000.00 p h p x 0.09 x =1,000.00 p h p
360

Principal’s Remainder = 80,000.00php – 1,000.00php = 79,000.00php


GENERAL MATHEMATICS 61 |
Page
New Balance = 200,000.00php – 79,000.00php =121,000.00php

40
Payment on Day 60: P = 121,000.00php, r = 9%, and t= (60 days−20 days=40 days)
360

40
I =Prt=121,000.00 p h p x 0.09 x =1,210.00 p h p
360

Total Final Payment Due = 1,210.00php + 121,000.00php = 122,210.00php

Principal’s Remainder = 122,210.00php – 1,210.00php = - 121,000.00php

New Balance = 121,000.00php – 121,000.00php = 0.00php

Note:

With partial payment: Without partial payment:

I =986.30+1,193.30 I =Prt
I =2,179.60 p h p 60
I =200,000 x 0.09 x
365
I =2,958.90 p h p

Activity #01

Vocabulary and Concepts

Ramil deposited 20,000.00php at 4% simple interest for 5 years. At the end


of 5 years, his account contains 24,000.00php. Give the term for each value in relation
to the problem.

GENERAL MATHEMATICS 62 |
Page
1. 20,000.00php 2. 24,000.00php 3. 4% 4. 5 years

Practice and Application

I. Complete the table by finding the simple interest.

Principal (P) Interest Rate (r) Time (t) Amount of Interest (I)

1. 72,500.00php 8% 3 years

2. 3,000.00php 12% 6 months

3. 1
48,200.00php 11% 2 years
2

4. 4
161,500.00php 10 % 4 years
5

5. 1
450,000.00php 9 % 5 years
4

II. Complete the table by finding the maturity value.

Principal (P) Interest Rate (r) Time (t) Amount of Interest (I)

1. 35,600.00php 6% 9 months

2. 140,250.00php 10% 15 months

3. 1
75,800.00php 8 % 2 years
2

4. 340, 200.php 11% 6 years

GENERAL MATHEMATICS 63 |
Page
5. 1,400,500.00 PHP 9% 10 years

Oronce, Orlando A. (2016), “Lesson 4.1:


Simple Interest”, GENERAL
MATHEMATICS, pp.186-195

Web and Book Links https://www.slideshare.net/hisema/


simple-and-compound-interest-24834757

GENERAL MATHEMATICS 64 |
Page
Lesson 2: Compound Interest

What to Know?

What you can learn and do in this lesson:

 Define compound interest.


 Illustrate compound interest.
 Distinguish between simple and compound interest.
 Compute interest, maturity value, future value, and present value in a simple
interest environment.
 Solve problems involving compound interest.

Pre – Test

Complete the table below showing the amount needed in any one of the first five years
to pay off a loan of 20,000.00php at 8% per year at simple interest.

Accumulated
At the end of Principal Amount Due
Interest

1 year 20,000.00php 1,600.00php 21,600.00php

2 years

3 years

4 years

5 years

Describe the data under the heading “Amount Due.”

GENERAL MATHEMATICS 65 |
Page
_______________________________________________________________________
_______________________________________________________________________
_______________________________________________________________________
_____________________

Let us Learn This!

The simple interest (I) on the principal (P) at the rate (r) per year for t years is

I =Prt

The simple interest on a loan of 20,000.00php at 8% for 3 years is

I =Prt =20,000 x 0.08 x 3=4,800

Thus, the interest is 4,800.00php at the end of 3 years for a loan of 20,000.00php at
8% simple interest.

Formula
Maturity Value (Amount or Balance)
A=P+ I or A=P+ Prt or A=P(1+rt)

A = Maturity Value P = Principal I = Interest

The total amount that must be repaid on a loan of 20,000.00php at 8% for 3 years is
24,800.00php.

A=P+ I

GENERAL MATHEMATICS 66 |
Page
A=20,000+4,800

A=24,800.00 p h p

Activity #01
 Arthur borrows 40,000.00php at 6% simple interest for a period of 1 year. At the end of
one year, how much must he pay?
 If Arthur did not pay back the loan or the interest by the end of the first year and wanted
to continue the loan for another year at the same rate, he would owe 40,000.00php plus
the interest incurred during the first year loan. How much he pays at the end of the
second year?
 If the same thing happens, Arthur could not pay back the loan or the interest by the end
of the second year. The lender gave him another year under the same condition. How
much must he pay at the end of the third year?

The problem in the activity is an example of compound interest. For example,


40,000.00php was loaned for 3 years with interest compound annually. Banks pay
compound interest on their savings accounts.

When the interest due at the end of a certain period is added to the principal,
and that sum earns interest for the next period, the interest paid is called compound
interest.

The example in the activity can be answered as shown in the following table:

Principal at Interest Amount


the start of

GENERAL MATHEMATICS 67 |
Page
(At the end of the
the year
year)

First 40,000.00php 40,000 x 0.06 x 1=2,400.00 40,000.00 p h p+2,400.00 p h p=42,400.00


Year

Second 42,400.00php 42,400 x 0.06 x 1=2,544.00 42,400.00 p h p+2,544.00 p h p=44,944.00


Year

Third 44,944.00php 44,944.00 x 0.06 x 1=2,696.64 44,944.00 p h p+2,696.64 p h p=47,640.64


Year

The amount (A) at the end of the year is equal to the sum of the principal (P)
and the interest (Pr) for that year. In symbols,

A=P+ Pr=P(1+r )

Thus,

First year : A=40,000 ( 1+0.06 )=42,400.00

Second year : A=42,400 ( r +0.06 )


¿ 40,000 ( 1+0.06 ) (1+ 0.06 )
2
¿ 40,000 ( 1+0.06 ) =44,944.00

Third year : A=44,944.00 ( 1+0.06 )


2
¿ 44,944.00 ( 1+0.06 ) ( 1+0.06 )
3
¿ 44,944.00 ( 1+0.06 ) =47,640.64

In general, when interest is compounded annually for n years, the amount (or
future value) A is

GENERAL MATHEMATICS 68 |
Page
t
A=P ( 1+r )

Note: Because interest is paid on interest, compound interest is always greater


than simple interest.

In the example, at 6% the simple interest on 40,000.00php is 2,400.00php per


year; hence, 7,200.00 for three years. After three years, the amount of compound
interest 7,640.64(47,640.64php – 40,000.00php = 7,640.64php) is 440.64php more
than the amount at simple interest.

Example:

Find the compound amount on deposit at the end of 1 year if 20,000.00php is


deposited at 4% compounded (a) annually and (b) semi-annually.

Solution:

t
a. Using the formula A=P ( 1+r ) where P = 20,000.00php, r = 4% or 0.04, and t = 1,
we have
A=20,000 ( 1+ 0.04 ) =20,800.00 p h p

b. A rate of 4% per year compounded semi-annually means a rate of 2% per half-year.

Principal at Amount
the start of Interest (At the end of the
the year year)

GENERAL MATHEMATICS 69 |
Page
First half 20,000.00php 1 20,000.00 p h p+ 400.00 p h p=20,400.00 p
20,000 x 0.04 x =400.00
2
year

Second 20,400.00php 1 20,400.00 p h p+ 408.00 p h p=20,408.00 p


20,400 x 0.04 x =408.00
2
half year

At the end of one year, the amount of deposit is 20,808.00php.

For example, 1b, the period (interval for compounding) is 6 months. There are
r
two periods per year: the rate is per period, and the number of periods in n is 2n.
2
thus, if the interest is compounded semi-annually for n years, the amount A is

( )
2t
r
A=P 1+
2

In the same way, if the interest is compounded quarterly, the rate per period is
r
, and there are 4n periods in n years. In symbols, the compound amount is
4

( )
2t
r
A=P 1+
4

In general, the pattern can be extended for compounding K times per year to get

( ) , where Kr is called the periodic rate.


Kt
r
A=P 1+
K

Example:

Identify the interest rate per compounding period and the number of
compounding periods for each of the following investments.
GENERAL MATHEMATICS 70 |
Page
a. 12% compounded monthly for 4 years.
b. 10.2% compounded quarterly for 9 quarters.
Solution:

r 0.12
a. r = 12% = 0.12 and K = 12 (months per year), then = =0.01 . The
12 12
number of compounding periods is 12t = 12 * 4 = 48.

r 0.102
b. r = 10.2% = 0.102 and K = 4 (quarters per year), then = =0.026. the
4 4
number of compounding periods is 9.

Activity #02

Vocabulary and Concepts

Ana deposits 100,00.00php in a savings account and leaves it for 5 years to


accumulate interest at 9% compounded semi-annually. Give the term for each value in
relation to the problem.

1. 100,000.00php
2. 5 years
3. 4.5%
4. 155,296.94php
5. 55,296.94php

Practice and Application

GENERAL MATHEMATICS 71 |
Page
I. Find the periodic rate if the rate is compounded (a) annually, (b) semi-annually,
(c) quarterly, (d) monthly, and (e) daily.

1. 10%
2. 8%
3. 12%
4. 11%
5. 15%

II. Find the future value of the indicated principal.

6. 30,000.00php at 8% compounded annually for 10 years.


7. 35,000.00php at 6% compounded semi-annually for 12 years.
1
8. 51,000.00php at 8 % compounded quarterly for 8 years.
2
3
9. 12,500.00php at 7 % compounded semi-annually for 12 years.
4
10. 152,000.00php at 9% compounded daily for 10 years.

Oronce, Orlando A. (2016), "Lesson 4.2:


Compound Interest", GENERAL
MATHEMATICS, pp.196-202
Web and Book Links

Sirug, Ph. D., Winston S. (2016), "Chapter


5: Simple and Compound Interests,
GENERAL MATHEMATICS FOR SENIOR
HIGH SCHOOL "A Comprehensive
GENERAL MATHEMATICS 72 |
Page
Approach K to 12 Curriculum Compliant",
pp. 159-160

https://www.slideshare.net/hisema/
simple-and-compound-interest-24834757

GENERAL MATHEMATICS 73 |
Page
Lesson 3: Simple Annuity

What to Know?

What you can learn and do in this lesson:

 Define annuity payment.


 Identify different types of annuity.
 Find the future value and present value of simple annuities.
 Solve problems involving consumer loans (amortization).

Pre – Test

Answer each of the following:

1. 50,000.00php is invested for 5 years at 8% compounded quarterly. Give the


n r
value of each variable in the formula A=P ( 1+i ) where i= and n = Kt.
K

(a) P =
(b) r =
(c) i =
(d) n =

2. if 10,000.00php is invested for 10 years at 6% compounded semi-annually,


the final amount is 18,061.11php. Give the value of each variable in the formula
n r
A=P ( 1+i ) where i= and n=Kt .
K

(a) P =
(b) r =
GENERAL MATHEMATICS 74 |
Page
(c) i =
(d) n =
(e) A =

Let’s Learn This!

Definition
Geometric Sequence
A sequence in which each term is obtained by multiplying the preceding term by a
fixed number (also known as the common ratio) is called a geometric sequence or a
geometric progression.

The following are geometric sequences:

 1, 3, 9, 27, 81, … with common ratio 3


1 1 1 1
 2 , 1, , , , … with common ratio
2 4 8 2
 x , x , x , x , x , … with common ratio x
2 3 4 5

 ( a+ b ) , ( a+b )2 , ( a+ b )3 , ( a+b )4 , … with ratio a + b


 ( 1.1 ) , (1.1 )2 , ( 1.1 )3 , ( 1.1 )4 , … with common ration 1.1

Definition
Geometric Series
The sum of the terms a geometric sequence is called a geometric series.

GENERAL MATHEMATICS 75 |
Page
Rule
The formula calculates the sum of a finite geometric sequence

a 1 ( 1−r n )
Sn =
1−r
Where a 1 is the first term, r is the common ratio, and n is the number of terms.

GENERAL MATHEMATICS 76 |
Page
Example:

Find the sum of the first 12 terms of the geometric sequence 3, -9, 27, -81, 243, …

Solution:

Given: a 1=3
r =−9 ÷ 3=−3

Solve for S12 by substituting n with 12, a 1 with 3, and r with -3 into the formula,

a 1 ( 1−r n )
Sn=
1−r

3 [ 1−(−3 )12 ]
S12=
1−(−3)

3 ( 1−531 441 )
S12=
1+3

3(−531 440)
S12=
4

S12=−398 580

GENERAL MATHEMATICS 77 |
Page
Activity #01
Mr. and Mrs. Mendoza are planning to have their own home but have a limited
budget. They went to a bank for advice on how they can produce enough amount for the
down payment on a house and lot they have chosen. This is the advice of the bank:

If you invest 20,000.00php at the end of each year for 5 years in an account that
pays interest at 10% compounded annually, you will have the amount for the down payment
of the house and lot at the end of 5 years.
House A: 850,740.00php
House B: 1,221,020.00php
House C: 2,110,000.00php

Down payment 10% of the price No Price Increase for the Next Five Years
A. The diagram below will help you answer the questions that follow

GENERAL MATHEMATICS 78 |
Page
1. Calculate the amount to which the first
20,000.00php will grow using the compound
interest formula.

2. Calculate the amount to which the second


20,000.00php will grow using the
compound interest formula.

3. Calculate the amount to which the third


20,000.00php will grow using the
compound interest formula.

4. Calculate the amount to which the fourth


20,000.00php will grow using the
compound interest formula.

5. Find the total of the separate


20,000.00php investment. Include the
fifth or last 20,000.00php in the sum.
GENERAL MATHEMATICS 79 |
Page
B. Enter the results from part (A) onto the following table.

Period Amount in Exponential Form Amount in Pesos

1 20 000 ( 1.1 )
4
29 282.00 p h p

TOTAL

C. Answer each.
1. what pattern is reflected in the middle column of the table? Discuss.
___________________________________________________________________________
___________________________________________________________
2. Using the pattern found in No. 1, calculate the sum of the amounts in the third column.

The activity involves a fixed payment for each period (20,000.00php at the end
of each year) and a fixed compound interest rate over a specified time (10%
compounded annually for 5 years).

If the payment for each period is fixed, and the compound interest rate is fixed
over a specified time, the payment is called an annuity payment. Accounts associated
with streams of annuity payments are called annuities. In the activity, the payment is
due at the beginning of each period, and the annuity is called an annuity due. Each
payment in an annuity is called the periodic payment. The time between the
successive payments dates of an annuity is called the payment interval. The interval

GENERAL MATHEMATICS 80 |
Page
between the beginning of the first payment period and the end of the last payment
period is called the annuity term.

The following are examples of annuities:

 Rent payments
 Pension
 Monthly payment of car loan or mortgage

The flowchart below gives the different kinds of annuities.

GENERAL MATHEMATICS 81 |
Page
Definition
Annuity – A fixed sum of money paid to someone at regular intervals, subject to a
fixed compound interest rate.
Annuity Certain – payable for a definite duration. Begins and ends on a definite or fixed date
(monthly payment of car loan).
Annuity Uncertain – annuity payable for an indefinite duration (example:
insurance): dependent on some certain event.
Simple Annuity – Interest conversation or compounding period is equal to or the same as
the payment interval.
General Annuity – interest conversion or compounding period is unequal or not the same as
the payment interval.

Ordinary Annuity ( A o ) – annuity in which the periodic payment is made at the end of each
payment interval.
Annuity Due – an annuity in which the periodic payment is made at the beginning of each
payment interval.
Deferred annuity – the periodic payment is not made at the beginning of the end of each
payment interval but later.
General Ordinary Annuity – the first payment is made at the end of every payment interval.
General Annuity Due – the first payment is made at the beginning of every payment
interval.
Perpetuities – a series of periodic payments which are to run infinitely or forever.

Examples:

Determine if the given situations represent simple annuity or general annuity.

a. Payments are made at the end of each month for a loan that charges 1.05%
interest compounded quarterly.

GENERAL MATHEMATICS 82 |
Page
b. A deposit of 5,500.00php was made at the end of every three months to an
account that earns 5.6% interest compounded quarterly.

Solution:

a. Since the payment interval at the end of the month is not equal to the
compounding interval. Quarterly, the situation represents a general annuity.

b. Since the payment interval at the end of every three months (or quarterly) is
equal to the quarterly compounding interval. The situation represents a simple
annuity.

The Activity in the Exploration can be translated into an annuity problem:


Suppose Mr. and Mrs. Mendoza deposits 20,000.00php at the end of each year for 5
years in an investment account that earns 10% per year compounded annually, what is
the amount of this annuity?

This is a simple ordinary annuity problem because payments are made at the
end of each year.

The annuity amount is the amount of down payment for the house and lot
chosen by the couple.

The annual interest rate r is 10% or 0.1. Because the interest is compounded
annually,

(
r =i=0.1 . i=
r 0.1
= =0.1
K 1 )

GENERAL MATHEMATICS 83 |
Page
The number of conversion periods n = 5.

We can use a table to organize the calculations.

Period Payment per Period Amount at 5 years

20 000 ( 1+0.1 ) or 29 282.00 p h p


4
1 20,000.00php

20 000 ( 1+0.1 ) or 26 620.00 p h p


4
2 20,000.00php

20 000 ( 1+0.1 ) or 24 200.00 p h p


4
3 20,000.00php

20 000 ( 1+0.1 ) or 22 200.00 p h p


4
4 20,000.00php

5 20,000.00php 20,000.00php

Total 122,102.00php

Note: The 20,000.00php deposited at the end of the first year will draw interest for
4
4 years to amount to 20 000 ( 1.01 ) . the 20,000.00php deposited at the end of the
3
second year will draw interest for 3 years so that it will amount to 20 000 ( 1.01 ) , and
so on.

Since most annuities involve relatively small periodic payments and longer
periods, they are affordable for the average person. If longer periods are involved, the
procedure we have done in the activity will be very tedious; hence, formulas are
needed to simplify computations of the future value of the annuity.

GENERAL MATHEMATICS 84 |
Page
Definition
The future value of an annuity is the total accumulation of the payments and interest
earned.
The present value of an annuity is the principal that must be invested today to provide the
regular payments of an annuity.

Note: The term future value that has been used with compound interest
means the same as used with an annuity: Future value comes at the end.

Simple Ordinary Annuity

Formula Formula

Future Value of Simple Ordinary Present Value of Simple Ordinary


Annuity Annuity

- The future value FV of simple - The present value PV of a simple


ordinary annuity is ordinary annuity is

P [ 1−( 1+i )−n ]


n
P∗( 1+i ) −1
FV = PV =
i i
Where
FV =Future Value∨ Amount ∈¿ Where
P=Periodic Payment PV =Present Value∨ Amount
i=interest rate per period . P=Periodic Payment
Where i=interest rate per period
r Where
i= ; r = annual rate, K =
K
r
i= ; r = annual rate, K =
number of conversion periods in a K
year number of conversion periods in a
GENERAL MATHEMATICS 85 |
Page
n=total number of conversion periods year
n=t∗K ; t=number of years n=total number of conversion periods
n=t∗K ; t=number of years

Using the formula for future of simple ordinary annuity to solve the activity in the

Exploration, we have P=20 000.00 p h p ,i=0.1 i= ( 10 %


1 ),∧n=5(n=5.1); thus,

n
P∗( 1+i ) −1
FV =
i

5
000∗( 1+0.1 ) −1
¿ 20
0.1

¿ 122 102.00 p h p

Thus, an investment of 20,000.00php at the end of 5 years at 10% compounded


annually would return 122,102.00php. The interest earned is 122,102.00 –
100,000.00php = 22,102.00php.

Example:

Rose works very hard because she wants to have enough money in her
retirement account when she reaches 60. She wants to withdraw 26,000.00php every 3
months for 20 years, starting 3 months after she retires. How much Rose's deposit at
retirement at 12% per year compounded quarterly for the annuity?

Solution:

The principal that Rose must deposit at retirement is the present value of the
annuity payments.
GENERAL MATHEMATICS 86 |
Page
Using the formula:
r 12
Write the values of P,i, and n. P=36 000.00 p h p , i= = %∨0.03 , and
K 4
n=t∗K=20 ( 4 ) =80

P [ 1−( 1+i )−n ]


Write the formula. PV =
i

36 000 [ 1−( 1+0.03 )−80 ]


Substitute for P, i, and n. PV =
0.03

Calculate. PV =1 087 227.48 p h p

Rose needs 1, 087, 227.48php at retirement to pay for the annuity.

Simple Annuity Due

Formula Formula

Future Value of Simple Annuity Due Present Value of Simple Annuity Due

P [ 1−( 1+i ) ] ∗( 1+i )


n −n
P∗( 1+i ) −1
FV = ∗(1+i ) PV =
i i

Where Where
FV =Future Value∨ Amount PV =Present Value∨ Amount
P=Periodic Payment P=Periodic Payment
i=interest rate per period . i=interest rate per period

GENERAL MATHEMATICS 87 |
Page
Where Where
r r
i= ; r = annual rate, K = i= ; r = annual rate, K =
K K
number of conversion periods in a number of conversion periods in a
year year
n=total number of conversion periods n=total number of conversion periods
n=t∗K ; t=number of years n=t∗K ; t=number of years

Example:

1. Suppose Mr. and Mrs. Mendoza deposited 20,000.00php at the beginning of each
year for 5 years in an investment that earns 10% per year compounded annually. What
is the amount or future value of the annuity?

Solution:

r 10 %
We know that P=20 000.00 p h p ,i= = =0.1 ,∧n=t∗K =5 (1 )=5.
k 1
Thus,
n 5
P∗( 1+i ) −1∗( 1+i ) 000.00∗( 1+0.1 ) −1
FV = =20 ∗(1+ 0.1 )=134 312.20
i 0.1

Note: An investment of 20,000.00php at the beginning of each year for 5 years


at 10% compounded annually would return 134, 312.20php. This is 12
210.20 more than when the investment is done at the end of each year.

2. Raul Jade borrows money to renovate her house and repays by making yearly
payments of 50,000.00php at the beginning of each year for 10 years at an interest
rate of 8% compounded annually. How much did Raul Jade borrow?

GENERAL MATHEMATICS 88 |
Page
Solution:

Given:
P=50,000.00 p h p
r 8%
i= = =0.08
K 1
n=t∗K=10 ( 1 )=10

P [ 1−( 1+i ) ] ∗( 1+i )


−n
PV =
i
50 000 [ 1−( 1+0.08 )−10 ]
¿ ∗( 1+ 0.08 ) Substitute the values of P, i, and
0.08
n.
¿ 362 344.40 p h p Calculate.

Raul Jade borrows 362, 344.40php.

Regular Payment of an Annuity

In the previous examples, we used the formulas:

1. FV =P [ ( 1+ i )n−1
i ] to solve the future value or amount of simple ordinary

annuity.

[ ]
n
1−( 1+i )
2. PV =P to solve the present value of the simple ordinary annuity.
i

3. FV =P [ ( 1+ i )−n−1
i ] (1+i) to solve the future value or amount of simple annuity

due, and

GENERAL MATHEMATICS 89 |
Page
[ ]
−n
1−( 1+i )
4. PV =P (1+i) to solve the present value of a simple annuity due.
i

Manipulating these equations, we can solve for the regular payment or periodic
payment (P), using the following formula:

REGULAR PAYMENT (P) OF AN ANNUITY

Formula Formula

Simple Ordinary Annuity Simple Annuity Due

( FV ) i ( FV ) i
P= P=
( 1+i )n−1 [ ( 1+i )n−1 ] ( 1+i )

( PV ) i ( PV ) i
P= −n P=
1− (1+i ) [ 1− (1+i )−n ] ( 1+i )
Where Where
FV =Future Value FV =Future Value
PV =Present Value PV =Present Value
P=Periodic Payment P=Periodic Payment
i=interest rate per period . i=interest rate per period .
Where Where
r r
i= ; r = annual rate, K = i= ; r = annual rate, K =
K K
number of conversion periods in a number of conversion periods in a
year year
n=total number of conversion periods n=total number of conversion periods
n=t∗K ; t=number of years n=t∗K ; t=number of years

Example:

GENERAL MATHEMATICS 90 |
Page
1. Mary borrows 500,000.00php to buy a car. She has two options to repay her loan.
The interest is compounded monthly.

Option 1: 24 monthly payments every beginning of the month at 12% per year.
Option 2: 60 monthly payments every end of the month at 15% per year.

Find:
a. Mary’s monthly payments under each option
b. The interest Mary pays under each option

Solution:

a. For option 1:

PV =500 000.00 p h p
r 12 %
i= = =0.01
K 12
n=24
P=?
( PV ) i
P= Regular payment (P) of a Simple Annuity Due
[ 1− (1+i )−n ] ( 1+i )
Formula
( 500 000 ) ( 0.01 )
¿
[ 1−( 1+0.01 )−24 ] ( 1+0.01 )

5 000
¿
0.214558211

¿ 23 303.70 p h p

GENERAL MATHEMATICS 91 |
Page
For option 2:

PV =500 000.00 p h p
r 15 %
i= = =0.0125
K 12
n=60
P=?

( PV ) i
P= −n Regular payment (P) of a Simple Ordinary Annuity Formula
1− (1+i )

( 500 000 ) ( 0.0125 )


¿ −60
1−( 1+0.0125 )

¿ 11894.97 p h p

b. The interest paid is the difference between the total amount paid and the principal
borrowed.

For option 1:
24 payments of 23, 303.70php: 24 x 23 303.70=559 288.80 p h p
Total interest paid is 559,288.80 p h p−500 000.00 p h p=59 288.80 p h p
Thus, the total interest paid is 59,288.80php.

For option 2:
60 payments of 11, 894.97php: 60 x 11894.97=713 698.20 p h p
Total interest paid is 713 698.20 p h p−500 000.00 p h p=213 698.20 p h p
Thus, the total interest paid is 213,698.20php.

GENERAL MATHEMATICS 92 |
Page
2. Aldous obtained a loan of 50,000.00php for the tuition fee of her son. She must
repay the loan by equal payments at the end of every six months for 3 years at 10%
interest compounded semi-annually. Find the periodic payment.

Solution:

PV =50 000.00 p h p r =10 %


r 10 %
t=3 years i= = =0.05
K 2
K=2 P=?
n=3 ( 2 )=6
( PV ) i
P= −n
1− (1+i )

( 50 000 ) ( 0.05 )
¿ −6
1−( 1+0.05 )

¿ 9 850.87 p h p

The gradual extinction of a loan over a period by means of a sequence of regular


or equal payments as to principal and interest due at the end of equal intervals of time
is known as amortization. In Example 2, we can say that the loan of 50,000.00php
that is amortized by equal periodic or installment payments of 9,850.87php at equal
intervals end of every six months becomes the present value of a simple annuity.

When a loan is gradually repaid, the construction of an amortization schedule is


very important for both the lender and the lendee. They will both see how much of
each payment goes to the interest and how much is applied to the reducing principal.

The Amortization Schedule

GENERAL MATHEMATICS 93 |
Page
Periodic Interest at Amount repaid Outstanding
payment at the 10% due at to the principal principal at the
end of every 6 the end of at the end of end of every 6
Period months every 6 months every 6 months
months.
(B) (C) (D) (E)
(A)
0 50,000.00php
1 9 850.87php 2 500.00php 7 350.87php 42 649.13php
2 9 850.87php 2 132.46php 7 718.41php 34 930.72php
3 9 850.87php 1 746.54php 8 104.33php 26 826.39php
4 9 850.87php 1 341.31php 8 509.56php 18 316.84php
5 9 850.87php 915.84php 8 935.03php 9 381.81php
6 9 850.87php 469.09php 9 381.81php 0.00php
Total 59 105.22php 9 105.24php 50 000.00php

The amortization schedule can be prepared as follows:

P 1. Calculate the periodic payment. Complete Column B with this periodic


payment.
R
2. To fill up Column C, calculate interest using the formula: i=Prt .
O

C
i=50 000 ( 0.10 ) ( 12 )=2 500.00 p h p for row 1
E i=42649.13 ( 0.10 ) ( 12 )=2 132.46 p h p for row 2

GENERAL MATHEMATICS 94 |
Page
D
i=34 930.72 ( 0.10 ) ( 12 )=1 746.54 p h p for row 3
U
i=26 826.39 ( 0.10 ) ( 12 )=1 341.31 p h p for row 4
R

E i=18 316.84 ( 0.10 ) ( 12 )=915.84 p h p for row 5


i=9 381.81 ( 0.10 ) ( 12 )=469.09 p h p for row 6
3. To fill up Column D, subtract Column C from Column B.

Column D :9 8 50.87−2 500.00=7 350.87 p h p for row 1

Column D :9 850.87−2 132.46=7 718.41 p h p for row 2

Column D :9 850.87−1 746.54=8 104.33 p h p for row 3

Column D :9 850.87−1 341.31=8 509.56 p h p for row 4

Column D :9 850.87−915.84=8 935.03 p h p for row 5

Column D :9 850.87−469.09=9 381.81 p h p for row 6

4. To fill up Column E, subtract Column D from Column E as follows:

Column E :50 000−7 350.87=32649.13 p h p

Column E : 42 649.13−7 718.41=34 930.72 p h p

GENERAL MATHEMATICS 95 |
Page
Colum E :34 930.72−8 104.33=26 826.39 p h p

Column E :26 826.39−8 509.56=18 316.84 p h p

Column E :18 316.84−8 935.03=9 381.81 p h p

C olumn E :381.81−381.81=0.00 p h p

Note:
1. The amount of the original loan is equal to the total repayments on the
principal.
2. The outstanding principal is equal to 0 at the end of the term.
Differences Between Business Loan and Consumer Loan

Business Loan Consumer Loan

Real estate, equipment,


furniture, fixtures,
1. Collateral inventory, or personal Real estate
assets of the business
owners

The business owners must Does not require a


2. Guarantor
sign the loan as guarantors guarantor

Requires credit report, tax


returns, and the last three Requires a credit report or
3. Documentation
years of financial tax returns
statements

Shorter and includes a Longer than the business


4. Terms
higher interest rate loan

5. Follow–up Annual reviews of the No further follow-up once


GENERAL MATHEMATICS 96 |
Page
relationship are often
the loan is released
conducted

Deferred Annuity

A deferred annuity is an annuity in which the first payment is not made at the
beginning nor the end of the payment interval but a later date. The length of time when
these payments are made is called the period of deferment. The first payment is
made one period after the period of deferment. Thus, annuity deferred for 6 periods
will have the first payment at the end of 7 periods. Likewise, in an annuity whose first
payment is made at the end of the 7 periods, the annuity is deferred for 6 periods.

Present Value and Future Value of Deferred Annuity

Formula
The present value PV of a deferred annuity is given by the formula:

PV =P [
1−( 1+i )−(n +d ) 1−( 1+ i )−d
i

i ]
Where
PV =Present Value
P=Regular payment

r
i=rate per conversion period(i= , w h ere r ist h e annual rate∧K ,is t h e number of conversion periods)
K
n=number of paying periods (n=t∗K , w h ere t is t h e number of years)
d=number of deferred periods

GENERAL MATHEMATICS 97 |
Page
Formula
The formula gives the present value FV of a deferred annuity:

FV =P [ ( 1+ i )n−1
i ]
Where
FV =Present Value
P=Regular payment

r
i=rate per conversion period(i= , w h ere r ist h e annual rate∧K is t h e number of conversion periods)
K
n=number of paying periods (n=t∗K , w h ere t is t h e number of years)

Note: The future value of a deferred annuity is the same as the future value of
simple ordinary annuity.

Example:

Find the present value of 10 semi-annual payments of 2,000.00 each if the first
payment is due at the end of 3 years and money is worth 8% compounded semi-
annually.

Solution:

Given:

P=2 000.00 p h p t=5


8%
r =8 % i= =0.04
2
K=2 n=5(2)=10
GENERAL MATHEMATICS 98 |
Page
To visualize and find d, we have:

d=5(see circles wit h x)

PV =?

[ ]
− ( n +d ) −d
1−( 1+i ) 1−( 1+ i )
PV =P −
i i

PV =2 000 [
1−( 1+0.04 )−(10 +5) 1−( 1+0.04 )−5
0.04

0.04 ]
PV =13 333.13 php

Oronce, Orlando A. (2016), “Lesson 4.3: Simple


Annuity”, GENERAL MATHEMATICS, pp.203-232

Web and Book


Links Sirug, Ph. D., Winston S. (2016), "Chapter 6: Simple
and General Annuities, GENERAL MATHEMATICS
FOR SENIOR HIGH SCHOOL "A Comprehensive

GENERAL MATHEMATICS 99 |
Page
Approach K to 12 Curriculum Compliant", pp. 161 -
196

https://www.slideshare.net/KrystenAmoranto/
simple-annuities-50463135

GENERAL MATHEMATICS 100 |


Page
Lesson 4: General Annuity

What to Know?

What you can learn and do in this lesson:

 Find the future value and present value of general annuities.

Pre – Test

To have an amount of 200,000.00php at the end of 12 years with an interest rate of


12% converted semi-annually, you must set aside 3,935.80php semi-annually.

Using the annuity problem above, give the value of each variable in the formula.

[ ]
−n
1−( 1+i )
PV =
i

P=¿ ¿ t=¿ ¿

r =¿ ¿ n=¿ ¿

K =¿ ¿ PV =¿ ¿

Let’s Learn This!

GENERAL MATHEMATICS 101 |


Page
Amount and Present Value of an Annuity

The present value of an annuity is the principal that must be invested today
to provide the regular payments of an annuity.

The amount of an annuity is the sum of the regular deposits plus interest.

The given situations involve the amount of an annuity:

 Resty saves 2,000.00php by making equal monthly payments at his bank.


 Donna plans to retrieve a millionaire by making equal semi-annual
deposits into her retirement saving plan.

The given situations involve the present value of an annuity:

 Ellen plans to repay her loan of 50,000.00php by making equally quarterly


payments.
 Samson plans to make equal monthly withdrawals from his
1,000,000.00php retirement income fund.

Simple and General Annuities

In a simple annuity, the interest conversion period is equal to or the same as


the payment interval.

A general annuity is an annuity wherein the interest conversion period is


unequal or not the same as the payment interval.

Exploration

Why is it important to start investing early to any savings plan?

Determine the amount of each annuity.

GENERAL MATHEMATICS 102 |


Page
A B

1. 5,000.00php deposited every 1. 5,000.00php deposited every


month for 5 years at 8% per year month for 5 years at 8% per year
compounded monthly. compounded semi-annually.

2. 1,500.00php deposited every six 2. 1,500.00php deposited every 5


months for 10 years at 6% per months for 10 years at 6% per
year compounded semi-annually. year compounded monthly.

3. 2,500.00php deposited every year 3. 2,500.00php deposited every year


for 15 years at 10% per year for 30 years at 10% per year
compounded annually. compounded semi-annually.

Compare the annuity problems in A to the annuity problems in B.

In Exploration, the statements in Column B have payment intervals that are not
the same as the compounding period; hence they are general annuities.

General Ordinary Annuity

FORMULA

Present Value of General Ordinary Future Value of General Ordinary


Annuity Annuity

[ ] [ ]
−n
1−( 1+i ) ( 1+i )n−1
PV =P FV =P
( 1+i )b−1 ( 1+ i )b−1

GENERAL MATHEMATICS 103 |


Page
Where

P=regular payment

(
i=rate per conversion period i=
r
K
; r=annual rate , K=No . of conversion periods∈a year )
n=number of conversion periods for t h e w h ole term (n=t∗K , w h ere t=term of an annuity )

p
b= ,where p is the number of months in a payment interval and c is the
c
number of months in a compounding period.

Example:

1. Find the present value of an ordinary annuity of 2,000.00php payable annually


for 9 years if the money is worth 5% compounded quarterly.

Solution:

Given: P=2,000.00 p h p

n=9∗4=36

r 5%
i= = =0.0125
K 4

c=3

p=12

p 12
b= = =4
c 3

GENERAL MATHEMATICS 104 |


Page
[ ]
−n
1−( 1+i )
PV =P
( 1+i )k −1

[ ]
−36
1−( 1+0.0125 )
PV =2 000
(1+ 0.0125 )4 −1

¿ 14 155.99 p h p

2. 25,000.00php will be invested I an account at the end of each year at 4%


compounded semi-annually. Find the size of the fund at the beginning of the 16 th year.

Solution:

P=25,000.00 p h p c=6

t=15 years p=12

p 12
K=2 b= = =2
c 6

r 4%
n=15 ( 2 )=30 i= = =0.02
K 2

FV =P
[
( 1+ i )−n−1
( 1+i )k −1 ]
FV =25 000
[ ( 1+0.02 )−30−1
( 1+0.02 )2−1 ]
¿ 502 080.19 p h p

GENERAL MATHEMATICS 105 |


Page
Present Value of General Annuity Due

FORMULA

Present Value of General Annuity Future Value of General Annuity Due


Due

FV =P [ ][
( 1+ i )n−1 i
+i
]
[ ][
b
i
]
1−( 1+i )
−n
i ( 1+i ) −1
PV =P b
+i
i ( 1+i ) −1

Where

P=annuity payment

i=rate per conversion period i= ( r


K
; r=annual rate , K=No . of conversion pe riods∈a year )
n=number of conversion periods for t h e w h ole term (n=t∗K , w h ere t=term of an annuity )

p
b= ,where p is the number of months in a payment interval and c is the
c
number of months in a compounding period.

Example:
GENERAL MATHEMATICS 106 |
Page
1. Find the present value of an annuity due of 10,000.00php payable quarterly for
10 years if money is worth 6% compounded semi-annually.

Solution:

P=10 000.00 p h p c=6

t=10 years p=3

p 3
K=2 b= = =0.5
c 6

r 6%
i= = =0.03
K 2

[ ][ ]
−n
1−( 1+i ) i
PV =P b
+i
i ( 1+i ) −1

[ ][ ]
−20
1−( 1+0.03 ) 0.03
PV =10 000 0.5
+0.03
0.03 ( 1+0.03 ) −1

PV =304 227.87

2. Micco wants to save 100,000.00php for her first year of college. She deposits
3,500.00php at the beginning of each month in an account that earns 4% per year
compounded semi-annually. Will Micco have enough money saved at the end of 2
years?

Solution:

Given:

P=3 500.00 p h p

n=t∗K=2∗2=4

GENERAL MATHEMATICS 107 |


Page
r 4%
i= = =0.02
K 2

c=6

p 1
b= =
c 6

FV =?

FV =P [
( 1+ i )n−1
i
i
][
b
( 1+i ) −1
+i
]
FV =3 500 [ ( 1+ 0.02 )4 −1
0.02 ][ 0.02
1
6
( 1+0.02 ) −1
+0.02
]
¿ 3 500 ( 4.12 ) ( 6.07 )

¿ 87 529.40 p h p

Because 87,529.40php is less than 100,000.00php, Micco will not have enough
money at the end of 2 years.

Regular Payment (P) of General Annuity

GENERAL MATHEMATICS 108 |


Page
To solve for P in the formula for PV and FV for General Annuity, we can
transform the formulas as follows:

For General Annuity,

P=FV
[ ( 1+ i )k −1
( 1+ i )n−1 ]
[ ]
k
( 1+i ) −1
P=PV −n
1−( 1+i )

Example:

1. Mr. and Mrs. Toledo will need 300,000.00php in 2 years to start their own
business. They plan t save money by making monthly deposits at the end of each
month in an account earning 8% per year compounded quarterly. How much must
they make monthly?

Solution:

FV =300 000.00 p h p

n=t∗K=2∗4=8

r 8%
i= = =0.02
K 4

p=1

GENERAL MATHEMATICS 109 |


Page
p 1
b= =
c 3

P=?

P=FV
[
( 1+ i )k −1
( 1+ i )n−1 ]
[ ]
1
3
( 1+0.02 ) −1
P=300 000
( 1+0.02 )8 −1

¿ 11574.16 p h p

2. A couple left their son with a 1,000,000.00php insurance policy. What monthly
income would the policy provide for 15 years if the insurance company pays 8%
compounded semi-annually?

Solution:

Given:

PV =1 000 000 000.00 p h p p=1

n=t∗K=15∗2=30 c=6

r 8% p 1
i= = =0.04 b= =
K 2 c 6

P=? P=PV
[
( 1+i )k −1
1−( 1+i )
−n
]

[ ]
1
6
( 1+0.04 ) −1
P=1 000 000 −30
1−( 1+0.04 )

¿ 9 481.53 p h p

GENERAL MATHEMATICS 110 |


Page
Activity #01

I. The present value of an annuity of 5,000.00php every end of 3 moths for 10


years when the interest rate is 4% compounded annually is 164,631.30php.

Use the given situation to give the values of the following variables:

1. P= __________ 6. i= __________
2. t= __________ 7. c= __________
3. K= __________ 8. p= __________
4. n= __________ 9. b= __________
5. r= __________ 10. PV = __________

II. Solve the following.


1. Find the amount of an annuity of 50,000.00php payable semi-annually for
5 years if money is worth 6% compounded quarterly.
2. What is the present value of a general annuity of 4200.00php per quarter
for 10 years if money is worth 9% compounded semi-annually?
1
3. If the interest is 8 % compounded annually, what is the present value of
2
7,500.00php payable at the end of each six months for 8 years?

Oronce, Orlando A. (2016), “Lesson 4.4: General


Annuity”, GENERAL MATHEMATICS, pp.233-242

Web and

GENERAL MATHEMATICS 111 |


Page
Book Links •https://www.investopedia.com/retirement/calculating-
present-and-future-value-of-annuities/

GENERAL MATHEMATICS 112 |


Page
Lesson 5: Stocks and Bonds

What to Know?

What you can learn and do in this lesson:

 Illustrates stocks and bonds


 Distinguish between stocks and bonds
 Describe the different markets for stocks and bonds
 Analyze the different markets for stocks and bonds
 Interpret the theory of efficient markets

Pre – Test

A. Calculate the gain or loss for each item and record your answer in the fourth
column under "Change in Price." In the fifth column, write whether the price change is
a gain or loss.

Item Price Bought Price Sold Change in Price Gain or Loss

1 1 445.20 p h p 1 480.00 p h p
2 496.80 p h p 490.80 p h p
3 215.60 p h p 240.80 p h p
4 1 193.20 p h p 1 150.00 p h p
5 379.20 p h p 600.00 p h p

B. Multiply the Number of Items by the Price Per Item to calculate the Total
Cost of the Items.

Item Number of Price per Item Total Cost of the


GENERAL MATHEMATICS 113 |
Page
Items Items
1 400 89.60 p h p
2 260 50.80 p h p
3 70 382.00 p h p
4 540 164.00 p h p
5 75 124.00 p h p

Let’s Learn This!

Definition
Commission – a payment to an agent or salesperson based on the value or quantity
of goods bought and sold: broker's or brokerage fee. A
commission is found by finding the percentage of a sale.

Example:

1. Tony sells T-shirts on weekends. He is paid a straight commission of 3% on his


sales. In April, his sales amounted to 15,000.00php. How much was his commission?

Solution:

Commission = 3% of the total sales

¿ 0.03 x 15 000.00 p h p=450.00

Tony received 450.00php commission.

2. Jisoo received a commission of 11,000.00php on a sale of 200,000.00php. What


was her rate of commission?

GENERAL MATHEMATICS 114 |


Page
Solution:

To get the rate of commission, divide the amount of commission by the amount of
sales.

11000 11
= =0.055∨5.5 %
200 000 200

3. Kirito receives a 6% commission on all items he sells. If his last week’s


commission was 7,500.00php, what were his total sales?

Solution:

To find Kirito’s total sales, divide the commission by the rate of commission

7 500
=125 000
0.06

Kirito's total sales were 125,000.00php.

Formula
The simple interest (I) on a principal (P) at an annual interest rate (r) for (t) years is
I =Prt
The future value (amount/maturity value) FV is the principal P and the interest I. Thus,
FV =P+ I =P+ Prt=P ( I +t )

Where
I =Interest
P=Principal
FV =Future Value , amount∨maturity value
r =annual rate of interest
t=number of years

GENERAL MATHEMATICS 115 |


Page
Definition
A loan is an arrangement in which a lender gives money (principal) to a borrower in
exchange for the future repayment of the principal along with interest or other finance
charges.

Loans and investments are considered a similar transactions in such a way that
they both involve the flow of money from one party to another, the return of money to
its source, and the payment of a fee to the source for the use of the money.

Take these two cases:

1. Deposit money in a savings account.


- The depositor may view it as an investment because interest may be
earned from the amount of money deposited. Still, the bank views it as
a loan from the depositor, which, in turn, the bank will lend to another
customer.
2. Borrowing money to buy a house from a bank.
- The borrower views the transaction as a loan, but the bank views it as
an investment to the borrower to make a profit.

Example:

Find the interest charged on a simple interest loan of 68,000.00php at a 12.5%


rate for 320 days.

Solution:

320
We are given P=68 000.00 p h p, r =12.5 %=0.125, and t=320 days= years .
365

Applying the Simple Interest Formula, we have

I =Prt

¿ ( 68 000 ) ( 0.125 ) ( 320


365 )

GENERAL MATHEMATICS 116 |


Page
¿ 7 452.05

The amount of interest is approximately equal to 7,452.05php.

Stock Valuation
This unit covers the computation for the current yield for a stock, the price-earning
ratio of stock, cost, and proceeds, and a stock's gain (or loss).

Definition
The current yield for a stock
To measure how much you have earned on a stock compared with other
investments, compute the current yield. The current yield is a way of determining the
current value of a stock. The current yield shows how much dived you can get as a
percentage of the stock's current price per share. If a stock pays no dividend, there is no
current yield. The current yield is computed using the formula:
Annual dividend per s h are
Current Yield=
current price per s h are

Example:

Suppose Paquito A. Kalibre University paid a dividend of 142.60php per share


last year. If yesterday's previous price was 2,300.00php, what is the current yield on
the stock?

Solution:

Given:

Annual dividend per s h are=142.60 p h p

Current price per s h are=2,300.00 p h p

GENERAL MATHEMATICS 117 |


Page
Annual dividend per s h are
Current Yield=
Current price per s h are

142.60
¿ =0.062=6.2 %
2,300.00

The current yield rate per share is 6.2%

Activity #01

Cai Lasaro is a holder of common stock, which he bought at 273.00php per share. If,
for the first year, he received a 45.00php dividend per share at the end of every 6
months, what is the annual yield rate on this stock?

Definition
Price-earnings Ratio of Stock
Another thing that some people use to help them decide which stock to buy is the
price-earning ratio. This ratio is found using the formula:
Current price per s h are
Price−earnings Ratio=
earnings per s h are

Example:

Francisco U. Balagtas University is currently selling for 2,685.00php per share. If


the university had earnings per share of 89.50php in the past year, what is the price-
earnings ratio for Francisco U. Balagtas University?

Solution:

Given:

Current price per s h are=2,685.00 p h p

GENERAL MATHEMATICS 118 |


Page
Earnings per s h are=89.50 p h p

Current price per s h are


Price−earnings Ratio=
Earnings per s h are

2,685.00
Price−earnings Ratio= =30∨30 :1
89.50

This means investors are currently willing to pay 30 times the earnings for one share of
Francisco U. Balagtas University stock.

Activity #02

1. Kawasaki Corp. is currently selling at 650.40php and the earning per share is
50.60php. What is the price-earnings ratio?

2. Waikiki Industries stock is currently selling at 2,385.00php. If the company had


earnings per share of 122.00php last year, determine the price-earning ratio of the
stock.

GENERAL MATHEMATICS 119 |


Page
Definition
Cost, Proceeds, and Gain (or loss) of a stock
Proceeds are the amount of money that an investor receives after selling a stock. It is
computed as the value of shares less the broker’s commission. The stockholder’s
commission is the fee charges for assisting in the purchase or sale of shares of stock, a
percent of the cost of the stock transaction. A stockbroker is a professional in stock market
trading and investment who acts as an agent in selling and buying stocks or other securities.
The gain (or loss) is the difference between the cost of purchasing the stock and the
proceeds received when selling the stock.
One more factor affecting the commission is whether the number of shares
purchased is a round lot (multiple of 100 shares) or an odd lot (less than 100 shares). The
commission rate on a round lot is generally a bit lower than an odd lot.
The following formula will be used:
Cost of s h ares=Price per s h are x Number of s h ares
'
Broker s Commission=Cost of s h ares x Commission rate
'
Total cost=Cost of s h ares+ Broke r s Comm i ssionrate
Value of s h ares=Price per s h are x Number of s h ares
'
Proceeds=Value of S h ares−Broker s Commission
Gain ( ¿ loss ) on transaction=Proce eds−Total cost

Example:

Riczel purchased 250 shares of AUS Inc. common stock at 3,500.00php per
share. A few months later, you sell the shares at 4,000.00php. Her stockbroker charges
3% commission on round lots and 4% on odd lots. Calculate the (a) total cost, (b) the
proceeds, and (c) the gain or loss on the transaction.

Solution:

Given: Price per s h are=3,500.00 p h p Commission rate ( round lots )=3 %=0.03

Number of s h ares=250 Commission rate ( odd lots )=4 %=0.04

GENERAL MATHEMATICS 120 |


Page
a. Cost of purchasing stock

Cost of s h ares=Price per s h are x Number of s h are=( 3,500 ) ( 250 )=875,000

'
Broker s commi ssion=Cost of s h ares x Commission rate

Round lot commission=250 s h ares x 3 500 x 0.03=21 000

Odd lot commission=50 s h ares x 3 500 x 0.04=7 000

'
Total Broke r s co mmission=21000+ 7 000=28 000

'
Total cost=Cost of s h ares+Total Broke r s commission

¿ 875 000+28 000=903 000

The total cost of 250 shares of common stocks is 903,000.00php.

b. Proceeds from selling stock

V alues of s h ares=Price per s h are x Number of s h ares

¿ 4 000 ( 250 )=1 000 000

'
Broker s commission=Cost of s h ares x Commission rate

Round lot commission=200 s h ares x 4 000 x 0.03=24 000

Odd lot commission=50 s h ares x 4 000 x 0.04=8000

'
Total Bro k e r s Commission=24 000+ 8 000=32 000

'
Proceeds=Value of s h ares−Broker s commission

¿ 1 000 000−32 000=968 000

The proceeds on common stock are 968,000.00php.

c. Gain (or loss) on the transaction

Gain ( ¿ loss ) on transaction=Proceeds−Total Cost

¿ 968 000−903 000=65 000.00 p h p

GENERAL MATHEMATICS 121 |


Page
The transaction gain 65,000.00php.

Activity #03

1. Lexi purchased 4,200 shares of YORME Corporation common stock at 7,500 per
share. A few months later, she sells the shares at 6,800.00php. Her stockbroker
charges 2.5% commission on round lots and 3.5% on odd lots. Calculate the
total cost, the proceeds, and the gain or loss on the transaction.

2. Johnny purchased a preferred stock at 292.00php per share. His monthly


dividend per share is 23.50php. Find the total gain per share if the investor sells
the stock after 3 years at 310.00php per share.

Bond Valuation
This unit covers the current yield of bonds and the price of bonds using the general
method.

Definition
Current Yield of Bond
The current yield of a bond is computed by dividing the annual interest by the bond's
purchase price. The current yield is obtained using the formula:
Annual I nterest
Current Yield=
Price of Bond

Example:

What is the current yield of a bond whose face value is 14,500.00php and pays a
yearly interest of 12% if purchased at a face value of 13,920.00php?

Solution:

GENERAL MATHEMATICS 122 |


Page
When the price of the bond is 14,500.00php

Annual Interest ( 0.12 ) ( 14 500 )


Current Yield= = =12 %
Price of Bond 14 500

When the price of bond is 13,920.00php

( 0.12 )( 14 500 )
Current Yield= =12.5 %
13 920

Activity #04

1. What is the current yield of a bond whose face value is 25,700.00php and pays a
yearly interest of 11.8% if purchased at face value of 24,120.00php?

2. Find the current yield of a bond whose face value is 13,150.00php and pays a
yearly interest of 6% if purchased at face value, 8,250.00php, and 6,350.00php?

GENERAL MATHEMATICS 123 |


Page
Definition
Price of a Bond using General Method
The same with stocks, when bonds are sold or bought, a charge is commonly added
to the price of the bond. The following variables will be in our mathematical treatment of
bonds:
V =Redemption value of a bond
F=Face value∨ par value of t h e bond
P=Price of t h e bond
r =Bond rate
k =Coupon payment
j=Nominal interest ra te
m=Number of conversions per year

t=Time period ( term ) of t h e loan∨investment


b=Periodic bond interest rate
i=Periodic interest rate

The regular (periodic) interest payment from the bond will be:

Coupon payment = Face value x Periodic bond interest rate

k =Fb

Time Diagram for Expected Payments from a Bond

GENERAL MATHEMATICS 124 |


Page
This figure shows the future payment that a potential investor can expect to receive.
The investor expects to receive n periodic interest payments of size Fb and, concurrent
with the final interest payment, Fb's value (par value of the bond).

The fair market value of a bond is the sum of the present interest payments and
the present value of the face value.

The price of a bond is computed using the formula below:

[ ]
−n
−n 1− (1+i )
P=V ( 1+i ) +k
i

Example:

A 3,200.00php at 9% bond pays coupons quarterly and will be redeemed on July


7, 2016. Find the price if the bond is bought on July 7, 2012, to yield 8% compounded
quarterly if the bond is redeemable at par, (b) if the bond is redeemable at 110%.

Solution:

Given: V =3,200.00 p h p j=8 %=0.08 t=4 years

m=4 r =9 %=0.09 n=tm=4 ( 4 )=16

r 0.09 j 0.08
b= = =0.0225 i= = =0.02
m 4 m 4

a. The redemption value is

k =Fb=3 200 ( 0.025 ) =72.00 p h p

The coupon payments are 72.00php.

[ ]
−n
1− (1+i )
P=V ( 1+i )−n +k
i

[ ]
−16
−16 1−( 1+0.02 )
P=3 200 ( 1+0.02 ) +72
0.02

¿ 3 308.62 p h p

GENERAL MATHEMATICS 125 |


Page
The price of the bond to yield 8% is 3,308.62php.

b. At 110% redemption means

V =3200 (1.10)=3 520

[ ]
−n
1− (1+i )
P=V ( 1+i )−n +k
i

[ ]
−16
−16 1−( 1+0.02 )
P=3 520 ( 1+0.02 ) +72
0.02

¿ 3 541.72 p h p

Activity #05

1. A 13,400.00php, 8% bond redeemable at 156.00php on January 13, 2017


paid interest quarterly. Find the price an investor would have been willing to
pay by January 13, 2020, if he wishes to earn 14% compounded semi-annually on his
investment?

Oronce, Orlando A. (2016), “Lesson 4.5:


Stocks and Bonds”, GENERAL
MATHEMATICS, pp.243-264
Web and Book Links

Sirug, Winston S. (2016, “Chapter 7: Stocks


and Bonds”, GENERAL MATHEMATICS for
senior high school: A Comprehensive
Approach, pp.197-216

GENERAL MATHEMATICS 126 |


Page

https://www.slideshare.net/pjgrethel/stocks-
bonds

GENERAL MATHEMATICS 127 |


Page
Lesson 6: Logic

What to Know?

What you can learn and do in this lesson:

 Illustrate a proposition.
 Symbolize propositions.
 Distinguish between simple and compound propositions.
 Perform different types of operations on propositions.

Pretest

Which sentence can be answered as either true or false?

1. Lester is passing in Mathematics.

2. Pass the paper and then leave the room.

3. December 7, 1953, was a Monday.

4. When will you submit your project?

5. 5 + 3 = 8 and 12 – 7 = 5.

Let’s Learn This!

GENERAL MATHEMATICS 128 |


Page
Definition
The term Logic is often used, but not always in its technical sense. Logic is
technically defined as "the science or study of how to evaluate arguments and reasoning."
Logic helps us to differentiate correct reasoning from poor reasoning. It is important from
poor reasoning. It is important in the sense that it allows us to reason correctly.

Mathematical Logic (or symbolic logic) is a branch of mathematics with close


connections to computer science. It includes both the mathematical study of logic and
the applications of formal logic to other areas of mathematics.

Definition
Proposition
A Proposition (or statement) is a declarative sentence that is either true or false, but
not both. The truth table of the propositions is the truth and falsity of the proposition.

Example:

Which of the following are propositions?

1. Manila is the capital of the Philippines.

2. What day is it?

3. Help me, please.

4. He is handsome.

Answer:

1. "Manila is the capital of the Philippines" is true. Therefore, it is a proposition.

2. “What day is it?” It is a question; it cannot be considered either true or false


and thus is not a proposition.

3. “Help me, please.” It cannot be categorized as true or false and therefore is


not a proposition.
GENERAL MATHEMATICS 129 |
Page
4. “He is handsome.” The sentence is neither true nor false because “he” is not
specified, and thus, it is not a proposition.

There are also propositions (or statements) that are considered ambiguous such as

1. Mathematics is fun.

2. Calculus is more interesting than Trigonometry.

3. It was hot in Manila.

4. Street vendors are poor.

Activity #01

Determine which of the following is the proposition:

1. 2+ 4=6 3. What is your name?

2. The world is flat. 4. It will snow tomorrow in Japan.

Definition
Simple and Compound Proposition
A propositional variable is a variable that is used to represent a proposition. A
formal propositional variable is written using propositional logic notation p, q, and r
represent propositions. Logical connectives are used to combine simple propositions, which
are referred to as compound propositions. A compound proposition is a proposition
composed of two or more simple propositions connected by logical connectives "and," “or”
"if-then," "not," "if and only if," and "exclusive-or." A proposition that is not compound is
said to be simple (also called atomic).

GENERAL MATHEMATICS 130 |


Page
Operations on Propositions

There are six main logical connectives: conjunction, disjunction, negation,


conditional, biconditional, and exclusive-or. The following are briefly discussed in this
section. Note that T refers to True and F refers to false proposition.

A. Conjunction – The conjunction of the proposition p and q is the compound


proposition "p and q." Symbolically, p ∧q , where ∧ is the symbol for
“and.”

Property 1: If p is true and q is true, p ∧q is true; otherwise, p ∧q is false.


Meaning, the conjunction of two propositions is true only if each proposition is
true.

p q p ∧q

T T T

T F F

F T F

F F F

Example:

1. 2+6=9, and man is a mammal.

- Since “2+6=9” is a false proposition (note that 2+6 ≠ 9) and the proposition
“man is a mammal” is true, the conjunction of the compound proposition is false.

2. Manny Pacquiao is a boxing champion, and Gloria Macapagal Arroyo is the first
female Philippine President.

- in the proposition "Manny Pacquiao is a boxing champion" is true. In contrast,


the proposition "Gloria Macapagal Arroyo is the first female Philippine President" is false

GENERAL MATHEMATICS 131 |


Page
(note that Corazon Aquino if the first female Philippine President). Therefore the
conjunction of the compound proposition is false.

3. Ferdinand Marcos is the only three-term Philippine President, and the Philippine
Senate is composed of 24 senators.

- Since both the propositions “Ferdinand Marcos is the only one three-term
Philippine President” and “Philippine Senate is composed of 24 senators” are
true. Thus the conjunction of the compound proposition is true.

Activity #02

Determine the truth values of each of the following conjunction.

1. The earth is a triangle, and the moon is a square.

2. Square has four sides, and the moon is square.

3. Apple is a fruit, and butterflies are insects.

B. Disjunction – the disjunction of the proposition p, q is the compound


proposition “p or q.” Symbolically, p ∨q , where ∨ is the symbol for "or."

Property 2: If p is true and q is true, p ∨q is true; otherwise, p ∨q is true.


Meaning, the disjunction of two propositions is false only if each proposition is
false.

p q p ∨q

T T T

T F T

F T T
GENERAL MATHEMATICS 132 |
Page
F F F

Example:

Determine the truth value of each of the following disjunctions.

1. 2+6=9 or Manny Pacquiao is a boxing champion.

- Note that the proposition “2+6=9” is false while the proposition “Manny
Pacquaio is a boxing champion” is true, hence the disjunction

2. Philippine Senate comprises 24 senators, or Gloria Macapagal Arroyo is the


first female Philippine President.

- Since the proposition "Philippine Senate is composed of 24 senators" is true,


the proposition "Gloria Macapagal Arroyo is the first female Philippine
President" is false. Therefore the disjunction of the compound proposition is
true.

3. Ferdinand Marcos is the only three-term Philippine President or man who is a


mammal.

- Given that both propositions, "Ferdinand Marcos is the only three-term


Philippine President" and "man is a mammal: are both true. Thus the
disjunction of the compound proposition is true.

Activity #03
Determine the truth values of each of the following disjunctions.

1. COBOL is programming or 2+5=7.

2. Windows 2010 is an operating system, or 15 is a prime number.


GENERAL MATHEMATICS 133 |
Page
3. 5 is less than 3 or 6 ÷ 1=6 .

C. Negation – the negation of the proposition p is denoted by p, where


is the symbol for “not.”

Property 3: If p is true, then pis false. Meaning, the truth value of the
negation of a proposition is always the reverse of the truth value of the original
proposition.

p p

T F

T F

F T

F T

Example:

The following are propositions for p. Find the corresponding p.

1. 3+5=8.

- 3+5 ≠ 8.

2. Sofia is a girl.

- Sofia is not a girl, or Sofia is a boy.

3. Achaiah is not here.

- Achaiah is here.

GENERAL MATHEMATICS 134 |


Page
Activity #04
The following are propositions for p. Find the corresponding p.

1. 23 is a composite number.

2. Earth has one moon.

3. Hexagon has five sides.

D. Conditional – the conditional (or implication) of the proposition p and q is


the compound proposition "if p then q." symbolically, p → q, where → is the symbol for
"if-then." p is called hypothesis (or antecedent or premise), and q is called
conclusion (or consequent or consequence).

Property 4: The conditional proposition p → qis false only when p is true, and q
is false; otherwise, p → q is true. Meaning p → q states that a true proposition
cannot imply a false proposition.

p q p→q

T T T

T F F

F T T

F F T

Example:

1. If vinegar is sweet, then sugar is sour.

- Since the propositions "vinegar is sweet" and the "sugar is sour" are both
false. Therefore the conditional of the compound proposition is true.

2. 2+5=7 is a sufficient condition for 5+6=1.


GENERAL MATHEMATICS 135 |
Page
- Note that “2+5=7” is true and “5+6=1" is false. Thus the conditional of the
compound proposition is false.

3. 14−8=4 is a necessary condition that 6 ÷ 3=2 .

- Given that “14−8=4 " is a false proposition (note that 14−8=6) while “6 ÷ 3=2 " is
a true proposition. Thus the conditional of the compound proposition is true.

Activity #05
Identify antecedent, consequent, and the truth value of each of the ff. conditional
propositions.

1. If a parallelogram has three sides, then 7 is a prime number.

2. The number 2 is even. Therefore 3 is an irrational number.

3. π is a rational number if 2 is less than 4.

E. Biconditional – The Biconditional of the proposition p and q is the compound


proposition “p if and only if q.” Symbolically, p ↔ q, where ↔ is the symbol for “if and
only if.”

Property 5: If p and q are true or both false, then p ↔ q is true; if p and q have
opposite truth values, then p ↔ q is false.

p q p↔q

T T T

T F F

F T F

F F T

GENERAL MATHEMATICS 136 |


Page
Example:

Determine the truth values of each of the following biconditional propositions.

1. 2+8=10 if and only if 6−3=3 .

- Since the statements “2+8=10” and the “6−3=3 " are both true, the compound
proposition's conditional is conditional.

2. Manila is the capital of the Philippines, which is equivalent to fish live on the
moon.

- Note that "Manila is the capital of the Philippines" is a true proposition while "fish
live in the moon" is false. Thus the conditional of the compound proposition is
false.

3. 8−2=5 is necessary and sufficient for 4 +2=7 .

- Given that “8−2=5” and “4 +2=7 ” are both false, thus the conditional of the
compound proposition is true.

Activity #06
Determine the truth value of each of the following biconditional propositions.

1. 8+ 4=5 if and only if 7 is less than 13.

2. 3 is a composite number equivalent to 11 is a prime number.

3. 6 is divisible by 3 is necessary and sufficient for 2+1=3 .

GENERAL MATHEMATICS 137 |


Page
F. Exclusive-or – The exclusive-or of the proposition p and q is the compound
proposition “p exclusive-or q.” Symbolically, p ⨁ q , where ⨁ is the symbol for
“exclusive-or.”

Property 6: If p and q are true or both false, then p ⨁ q is false; if p and q have
opposite truth values, then p ⨁ q is true.

p q p⨁q

T T F

T F T

F T T

F F F

It can be noted that the truth values of p ⨁ q are the negation of the truth
values of p ↔ q.

Given the proposition "Sofia will take her lunch in Batangas, or she will have it in
Singapore," it can be noted from the statement that "Sofia cannot have her lunch in
Batangas and at the same time do it in Singapore," thus it is considered false.

Lastly, if she decides to have her lunch elsewhere (neither in Batangas nor
Singapore), the truth value is false.

The statement formula without parenthesis that contains logical connectives is


evaluated in the following order:

(Top Priority)

∧(Second Priority)

∨(Third Priority)

→(Fourth Priority)

GENERAL MATHEMATICS 138 |


Page
↔(Fifth Priority)

Oronce, Orlando A. (2016), “Lesson 5.1:


Propositions and Symbols”, GENERAL
MATHEMATICS, pp.266-277

Sirug, Winston S. (2016, “Chapter 9: Logic”,


GENERAL MATHEMATICS for senior high school:
Web and Book
A Comprehensive Approach, pp.-237-244
Links

•https://www.slideshare.net/AdilAslam4/chapter-
1-propositions-in-discrete-mathematics

GENERAL MATHEMATICS 139 |


Page
Lesson 7: Truth Values of Propositions

What to Know?

What you can learn and do in this lesson:

 Determine the truth values of propositions.


 Illustrate the different forms of conditional propositions.

Pretest

Construct the truth table for

1. p∨ q

2. ( p ∧q )

3. q ∧ ( p ∨ q)

Let’s Learn This!

This section shows the construction of compound propositions through truth


tables. Let us examine the examples below.

Example:

Construct the truth table for each of the following propositions.

1. p∨ q

p q ~p ~q p∨ q

GENERAL MATHEMATICS 140 |


Page
T T F F F

T F F T T

F T T F T

F F T T T

We need to determine the ~p and ~q before obtaining the truth values of p ∨ q .

2. p →( p ↔ q)

p q ~q ( p↔ q) p →( p ↔ q)

T T F F F

T F T T T

F T F T T

F F T F T

3. ( p ∧q ) ∨ ( p ∧ q )

p q p ( p ∧q ) q ( p∧ q) ( p ∧q ) ∨ ( p ∧ q )

T T F F F F F

T F F F T T T

F T T T F F T

F F T F T F F

4. ( p → q) ∨ ( q ∧ r )

p q r ( p → q) r (q ∧ r ) ( p → q) ∨ ( q ∧ r )

GENERAL MATHEMATICS 141 |


Page
T T T T F F T

T T F T T T T

T F T F F F F

T F F F T F F

F T T T F F T

F T F T T T T

F F T T F F T

F F F T T F T

Activity #01
Construct the truth table of the following compound propositions.

1. [ ( p ∧q ) ∨ ( p ∨ q ) ]

2. ( p ↔ q ) ⨁ ( p→ q )

3. p ∧(q ∨ r )

Oronce, Orlando A. (2016), “Lesson 5.2: Truth


Tables”, GENERAL MATHEMATICS, pp.278-289

Web and Book


Links Sirug, Winston S. (2016, “Unit 9.4: Truth Values
of Propositions”, GENERAL MATHEMATICS for
senior high school: A Comprehensive Approach,
pp.-245-247

GENERAL MATHEMATICS 142 |


Page
•https://www.slideshare.net/AdilAslam4/chapter-
1-propositions-in-discrete-mathematics

GENERAL MATHEMATICS 143 |


Page
Lesson 8: Tautologies, Contradiction, and
Contingency

What to Know?

What you can learn and do in this lesson:

 Illustrate the different types of tautologies and fallacies.


 Determine the validity of categorial syllogisms.
 Establish the validity and falsity of real-life arguments using logical propositions,
syllogisms, and fallacies.

Pretest

Table 1 T F T

Truth table for the Conjunction p and q. F T T

p q p ∧q F F F

T T T

T F F

F T F Table 3

F F F Truth table for the negation.

Table 2

Truth table for the disjunction p or q.

p q p ∨q

T T T

GENERAL MATHEMATICS 144 |


Page
p q p ∨q

T T T

T F T

F T T

F F F

GENERAL MATHEMATICS 145 |


Page
Let’s Learn This!

There are three important compound statements, namely tautology,


contradiction, and contingency, briefly discussed below.

A. Tautology – is a compound statement that is true for all possible combinations


of the truth values of the propositional variables, also called logically true.

B. Contradiction – is a compound statement that is false for all possible


combinations of the truth values of its propositional variables, also called logically
false or absurdity.

C. Contingency – is a compound statement that can be either true or false,


depending on the truth values of the propositional variables, is neither a tautology nor a
contradiction.

Example:

Write the truth table for each of the following compound statements and determine
whether the compound statement is a tautology, contradiction, or contingency.

1. ( p ∧q ) → q

p q ~p ( p ∧q ) ( p ∧q ) → q

T T F F T

T F F F T

F T T T T

F F T F T
Since all the truth values of the compound statement ( p ∧q ) → q are true, thus it is a
tautology.

2. ( p → q) ∧ ( p → q )

p q ( p → q) q ( p→ q) ( p → q) ∧ ( p → q )

T T T F T F

T F F T T F

F T T F T T

F F T T T T

Note that the truth values of the statement ( p → q ) ∧ ( p → q ) are combinations of true
and false. Therefore the compound statement is contingency.

3. ( p ∨q ) ⨁ ( p → q )

p q p ( p ∨q ) ( p → q) ( p ∨q ) ⨁ ( p → q )

T T F T T F

T F F F F F

F T T T T F

F F T T T F

Observe that all the truth values of the compound statement are false. Thus it is a
contradiction.
Activity #01
Construct the truth tables for the following and determine whether the compound
proposition is a tautology, contradiction, or contingency.

1. p ⨁ ( p ↔q )

2. [ p ∧ ( p →q ) ] →q

3. p → ( q ↔r )

Logical Equivalence

This section covers the definition of logically equivalent propositions and the laws
of logical equivalent, which were supported with some examples.

Logically Equivalent – Two propositions are said to be logically equivalent (or


equivalent) if they have the same value for every row of the truth table, that is, if
x ↔ y is a tautology. Symbolically, x ≡ y .

Example:

1. p ∧ ( q ∨ r ) and ( p ∧q ) ∨ ( p ∧ r )

p q r (q ∨ r ) p ∧(q ∨ r ) ( p ∧q ) ( p ∧r ) ( p ∧q ) ∨ ( p ∧ r )

T T T T T T T T

T T F T T T F T

T F T T T F T T

T F F F F F F F
F T T T F F F F

F T F T F F F F

F F T T F F F F

F F F F F F F F

Observe that the truth values of compound statements p ∧ ( q ∨ r ) and ( p ∧q ) ∨ ( p ∧ r ) are


the same. Thus we can say that they are logically equivalent.

2. p → q and q → p

p q p→q q p q→ p

T T T F F T

T F F T F F

F T T F T T

F F T T T T

Notice that the truth values of compound statements p → q and q → p are equivalent;
therefore, we can state they are logically equivalent.

3. p ↔ q and ( p → q ) ∧ ( q → p )

p q p↔q ( p → q) ( q → p) ( p → q) ∧ ( q → p)

T T T T T T

T F F F T F
F T F T F F

F F T T T T

The truth values of p ↔ q and ( p → q ) ∧ ( q → p ) are equal. Thus we can say that they are
logically equivalent.

Activity #02
Determine whether the following statements are logically equivalent using truth tables.

1. ( p ∨q ) and p ∧ q

2. ( p ∧q ) → r and p ↔ ( q →r )

3. p ↔ q and ( p ∨q ) ∧ ( p ∨ q )

Laws of Logical Equivalence – Let p, q, and r stand for any propositions. Let T indicate
tautology and F indicate contradiction.
Laws of Logical Equivalence

Laws Logical Equivalence

Communication p ∧q ≡ q ∧ p

p ∨q ≡ q ∨ p

Associative p ∧ ( q ∧ r ) ≡ ( p ∧q ) ∧ r

p ∨ ( q ∨ r ) ≡ ( p ∨q ) ∨ r

Distributive p ∧ ( q ∨ r ) ≡ ( p ∧q ) ∨ ( p ∧ r )

p ∨ ( q ∧ r ) ≡ ( p ∨q ) ∧ ( p ∨ r )

Identity p ∧T ≡ p

p∨F≡ p

Inverse/Negation p ∧∼ p ≡ F

p ∨∼ p ≡T

Double Negation ( p) ≡ p

Idempotent p∧ p≡ p

p∨ p≡ p

De Morgan’s ( p ∧q ) ≡ p ∨ q

( p ∨q ) ≡ p ∧ q

Universal Bound/Domination p∧F≡F

p ∨T ≡ T

Absorption p ∧ ( p ∨q ) ≡ p

p ∨ ( p ∧q ) ≡ p
Example:

Simplify the following compound statements using the laws of equivalence.

1. [ p ∨ ( p ∧q ) ] ∨ ( p ∨ q )

[ p ∨( p ∧q ) ] ∨ ( p ∨ q ) ≡ [ ( p ∨ p ) ∧ ( p ∨q ) ] ∨ ( p ∨ q ) Distributive Law

[ p ∨( p ∧q ) ] ∨ ( p ∨ q ) ≡ [ T ∧ ( p ∨ q ) ] ∨ ( p ∨ q ) Inverse Law

[ p ∨( p ∧q ) ] ∨ ( p ∨ q ) ≡ ( p ∨q ) ∨ ( p ∨ q ) Identity Law

[ p ∨( p ∧q ) ] ∨ ( p ∨ q ) ≡ p ∨ ( q ∨ q ) Distributive Law

[ p ∨( p ∧q ) ] ∨ ( p ∨ q ) ≡ p ∨T Inverse Law

[ p ∨( p ∧q ) ] ∨ ( p ∨ q ) ≡T Universal Bound Law

2. [ q ∨ ( p ∨ q ) ∨ ( p ∧q ) ] ∧ q ≡ [ q ∨ ( q ∨ p ) ∨ ( p ∧ q ) ] ∧ q Commutative Law

[ q ∨ ( p ∨ q) ∨ ( p ∧q ) ] ∧ q ≡ [ ( q ∨ q ) ∨ p ∨ ( p ∧ q ) ] ∧ q Associative Law

[ q ∨ ( p ∨ q) ∨ ( p ∧q ) ] ∧ q ≡ [ T ∨ p∨ ( p ∧ q ) ] ∧ q Inverse Law

[ q ∨ ( p ∨ q) ∨ ( p ∧q ) ] ∧ q ≡ [ ( T ∨ p ) ∨ ( p ∧ q ) ] ∧ q Associative Law

[ q ∨ ( p ∨ q) ∨ ( p ∧q ) ] ∧ q ≡ [ T ∨ ( p ∧ q ) ] ∧ q Universal Bound

[ q ∨ ( p ∨ q) ∨ ( p ∧q ) ] ∧ q ≡T ∧ q Universal Bound

[ q ∨ ( p ∨ q) ∨ ( p ∧q ) ] ∧ q ≡ q Identity Law

3. ( p ∧ q ) ∧ ( p ∨ q ) ∧ ( p ∨ q ) ≡ ( p ∧ q ) ∧ ( p ∨ q ) ∧ ( p∨ q ) Commutative Law

( p ∧ q )∧ ( p ∨ q) ∧ ( p ∨ q ) ≡ [ ( p ∧ q) ∧ ( p ∨ q ) ] ∧ ( p ∨ q ) Commutative Law
( p ∧ q ) ∧ ( p ∨ q ) ∧ ( p ∨ q ) ≡ { p ∧ [ q ∧ ( p ∨ q ) ]} ∧ ( p ∨ q ) Associative Law

( p ∧ q ) ∧ ( p ∨ q ) ∧ ( p ∨ q ) ≡ ( p ∧ q ) ∧ ( p ∨q ) Absorption Law

( p ∧ q ) ∧ ( p ∨ q ) ∧ ( p ∨ q ) ≡ p ∧ [ q ∧ ( p ∨q ) ] Associative Law

( p ∧ q )∧ ( p ∨ q) ∧ ( p ∨ q ) ≡ p ∧ [ ( q ∧ p) ∨ ( q ∧ q ) ] Distributive Law

( p ∧ q )∧ ( p ∨ q) ∧ ( p ∨ q ) ≡ p ∧ [ ( q ∧ p) ∨ F ] Inverse Law

( p ∧ q )∧ ( p ∨ q) ∧ ( p ∨ q ) ≡ p ∧ ( q ∧ p) Identity Law

( p ∧ q )∧ ( p ∨ q) ∧ ( p ∨ q ) ≡ p ∧ ( p ∧ q) Commutative Law

( p ∧ q )∧ ( p ∨ q) ∧ ( p ∨ q ) ≡ ( p ∧ p ) ∧ q Associative Law

( p ∧ q )∧ ( p ∨ q) ∧ ( p ∨ q ) ≡ F ∧ q Inverse Law

( p ∧ q )∧ ( p ∨ q) ∧ ( p ∨ q ) ≡ q Universal Bound

Activity #03
Simplify the following compound statements using the laws of equivalence.

1. ( p ∨ q ) ∧ [ p ∨ ( p ∧ q ) ] 2. ( p ∧q ) ∨ ( p ∧ q )

Oronce, Orlando A. (2016), "Lesson 5.3: "Forms of


Conditional Proposition," GENERAL MATHEMATICS,
pp.290-303
Web and
Book Links

Sirug, Winston S. (2016, "Unit 9.6: "Tautologies,


Contradiction, and Contingency," GENERAL
MATHEMATICS for senior high school: A Comprehensive
Approach, pp.247-251

•https://www.slideshare.net/forwardblog4u/
propositional-logic-14203172

You might also like